Google Groups no longer supports new Usenet posts or subscriptions. Historical content remains viewable.
Dismiss

Das sieht ja traurig aus hier

596 views
Skip to first unread message

Ganzhinterseher

unread,
May 16, 2022, 1:13:40 PM5/16/22
to
Um die positiven Brüche zu indizieren, prüfen wir zunächst die Menge der Indizes, indem wir sie mit den Ganzzahlbrüchen der ersten Spalte der Matrix
1/1, 1/2, 1/3, 1/4, ...
2/1, 2/2, 2/3, 2/4, ...
3/1, 3/2, 3/3, 3/4, ...
4/1, 4/2, 4/3, 4/4, ...
5/1, 5/2, 5/3, 5/4, ...
...
in Bijektion setzen. Dann wenden wir die Indizes nach Cantors Vorschrift an. Der Bruch m/n erhält den Index
k = (m + n - 1)(m + n - 2)/2 + m
mit dem Ergebnis, dass die Brüche der Folge
1/1, 1/2, 2/1, 1/3, 2/2, 3/1, 1/4, 2/3, 3/2, 4/1, 1/5, ...
alle indiziert werden. Die anfangs durchnummerierten Ganzzahlbrüche müssen diese Indizes liefern. (Sie werden später von weiter unten entschädigt.) Jeder neu indizierte Bruch verursacht den Verlust eines indizierten Ganzzahlbruches.

Kann jemand dieses Argument nicht verstehen?

Gruß, WM

Stefan Schmitz

unread,
May 16, 2022, 1:25:36 PM5/16/22
to
Am 16.05.2022 um 19:13 schrieb Ganzhinterseher:
> Um die positiven Brüche zu indizieren,

brauchen normale Mathematikstudenten wenige Minuten.
Ein Prof aus Augsburg kriegt das in Jahrzehnten nicht hin.

Ganzhinterseher

unread,
May 16, 2022, 3:15:44 PM5/16/22
to
Stefan Schmitz schrieb am Montag, 16. Mai 2022 um 19:25:36 UTC+2:
> Am 16.05.2022 um 19:13 schrieb Ganzhinterseher:
> > Um die positiven Brüche zu indizieren,
> brauchen normale Mathematikstudenten wenige Minuten.

Hast Du das Argument verstanden?

Gruß, WM

Juergen Ilse

unread,
May 16, 2022, 11:31:12 PM5/16/22
to
Hallo,
Ich verstehe davon nur, dass SIE offenbar eine voellig krude und
inkonsistente Vorrstellung von Unendlichkeit haben und ohne den
geringsten Beweis fuer die Zulaessigkeit der Uebertragungvon
einfch Eigenschaften von endlichen Mengen auf unendliche Mengen
uebertragen. Daraus folgen dann falsche Argumente und unzutref-
fende Beweisversuche, und SIE sind zu unfaeehig, um die Fehler
darin zu erkennen. So setzen SIE z.B. die (nur fuer endliche
Mengen zutreffende) Aussage implizit voraus, dass wenn zwischen
2 Mengen A und B *eine* bijektive Abbildung existiert, auch *jede*
*andere* Abbildung zwischen den beiden Mengen bijektiv sein muesse
(SIE reden/schreiben an den entsprechenden Stellen von "aufbrauchen
vo Elementen fuer die Indizierung" oder "Meneg n, die sich erschoepfen"
oder aehnliches). SIE bemerken dabein gar nicht, an welchen Stellen
SIE unzutreffende Eigenschaften von unendlichen Mengen voraussetzen,
nur weil diese fuer *endliche* Mengen zutreffend sind. Und um die
Inkonsistenzen zu uebertuenchen, erfinden SIE dann so seltsame nicht
existierende Dinge wie "potentiell unendliche Mengen" oder "dunkle
Zahlen" ...

Tschuess,
Juergen Ilse (jue...@usenet-verwaltung.de)
PS: Nein, es gibt weder "potntiell unendliche Mengen" noch "dunkle
Zahlen" ...

Gus Gassmann

unread,
May 17, 2022, 6:16:44 AM5/17/22
to
Es sieht echt traurig aus um jeden, der deinen obigen Scheissdreck für ein Argument hält.

JVR

unread,
May 17, 2022, 7:44:25 AM5/17/22
to
Ich hatte schon gehofft, dass Sie dauerhaft verschwunden sind.

Haben Sie doch keine neue Spielwiese gefunden, wo Sie nicht sofort wieder gesperrt werden?

Seit wie vielen Monaten wiederholen Sie jetzt schon Ihre Fehlüberlegung zur
ersten Spalte der Matrix
> 1/1, 1/2, 1/3, 1/4, ...
> 2/1, 2/2, 2/3, 2/4, ...
> 3/1, 3/2, 3/3, 3/4, ...
> 4/1, 4/2, 4/3, 4/4, ...
> 5/1, 5/2, 5/3, 5/4, ...
??

Ganzhinterseher

unread,
May 17, 2022, 8:30:38 AM5/17/22
to
Juergen Ilse schrieb am Dienstag, 17. Mai 2022 um 05:31:12 UTC+2:
> So setzen SIE z.B. die (nur fuer endliche
> Mengen zutreffende) Aussage implizit voraus, dass wenn zwischen
> 2 Mengen A und B *eine* bijektive Abbildung existiert, auch *jede*
> *andere* Abbildung zwischen den beiden Mengen bijektiv sein muesse

Ich setze hier nichts dergleichen voraus. Ich setze lediglich voraus, dass es genau so viele natürliche Zahlen wie positive Ganzzahlbrüche gibt. Was ist dagegen einzuwenden? (Bevor Du antwortest versuche erst einmal das zu verstehen.)

> SIE bemerken dabein gar nicht, an welchen Stellen
> SIE unzutreffende Eigenschaften von unendlichen Mengen voraussetzen,
> nur weil diese fuer *endliche* Mengen zutreffend sind.

Wenn man die Elemente einer Menge sammelt und für jedes eingesammelte Element ein vorher eingesammeltes wieder wegwirft, so kann man die Sammlung nicht vervollständigen. Das gilt wohl für endliche wie für unendliche Mengen.

> Und um die
> Inkonsistenzen zu uebertuenchen, erfinden SIE dann so seltsame nicht
> existierende Dinge wie "potentiell unendliche Mengen" oder "dunkle
> Zahlen" ...

Welche Inkonsistenz meinst Du entdeckt zu haben?

Gruß, WM

JVR

unread,
May 17, 2022, 8:41:09 AM5/17/22
to
Ein Fehler (es bei weitem nicht der einzige) kommt daher, dass Sie den undefinierten
Begriff "genau so viele" unkritisch verwenden.
Ganz analog: Ihr Missverständnis des Banach-Tarski Theorems kommt daher, dass Sie
den Begriff 'Volumen' falsch deuten.
usw usw - ein hoffnungsloser Fall ist dieser Prefosser.

Ganzhinterseher

unread,
May 17, 2022, 8:41:47 AM5/17/22
to
Was findest Du denn falsch?

Gruß, WM

Ganzhinterseher

unread,
May 17, 2022, 8:44:53 AM5/17/22
to
JVR schrieb am Dienstag, 17. Mai 2022 um 13:44:25 UTC+2:

> Seit wie vielen Monaten wiederholen Sie jetzt schon Ihre Fehlüberlegung zur
> ersten Spalte der Matrix
> > 1/1, 1/2, 1/3, 1/4, ...
> > 2/1, 2/2, 2/3, 2/4, ...
> > 3/1, 3/2, 3/3, 3/4, ...
> > 4/1, 4/2, 4/3, 4/4, ...
> > 5/1, 5/2, 5/3, 5/4, ...
> ??

Die Bijektion der natürlichen Zahlen mit den Ganzzahlbrüchen ist eine Fehlüberlegung?

Gruß, WM


Ganzhinterseher

unread,
May 17, 2022, 8:48:42 AM5/17/22
to
JVR schrieb am Dienstag, 17. Mai 2022 um 14:41:09 UTC+2:

> Ein Fehler (es bei weitem nicht der einzige) kommt daher, dass Sie den undefinierten
> Begriff "genau so viele" unkritisch verwenden.

Die Mengenlehre erfordert also dass man nicht genau so viele natürliche Zahlen wie Ganzzahlbrüche annehmen darf? Das sollte man den Erstsemestern freilich nicht mitteilen, sonst wären die meisten wohl gleichzeitig Letztsemester.

Gruß, WM

JVR

unread,
May 17, 2022, 9:50:33 AM5/17/22
to
Als Lehrer, wenn Sie einer wären (damit meine ich jemand, der 'Wissen vermittelt', nicht jemand wie Sie, der
Abgeschriebenes repetiert), wären Sie also der Frage ausgesetzt, wie es denn sein kann, dass es
halb so viele gerade Zahlen wie Ganze geben kann, wenn es doch offensichtlich (n <-> 2n) gleich viele sind.

Fritz Feldhase

unread,
May 17, 2022, 9:55:36 AM5/17/22
to
On Monday, May 16, 2022 at 7:13:40 PM UTC+2, Ganzhinterseher wrote

M1:

> 1/1, 1/2, 1/3, ...
> 2/1, 2/2, 2/3, ...
> 3/1, 3/2, 3/3, ...
> 4/1, 4/2, 4/3, ...
> ...

Diese ist wie folgt definiert:

M1_n,m = n/m für alle n,m e IN.

Ja, schön. Man kann zudem auch die Matrix M2 betrachten:

1/1, 1/2, 1/4 ...
1/3, 1/5, 1/8 ...
1/6, 1/9, 1/13 ...
1/10, 1/14, ...
...

Diese ist wie folgt definiert:

M2_n,m = 1/[(m + n - 1)(m + n - 2)/2 + m] für alle n,m e IN.

Was genau verstehen Sie an diesem trivialen Sachverhalt nicht?
Message has been deleted

Juergen Ilse

unread,
May 17, 2022, 10:07:46 AM5/17/22
to
Hallo,

Ganzhinterseher <askas...@gmail.com> wrote:
> Juergen Ilse schrieb am Dienstag, 17. Mai 2022 um 05:31:12 UTC+2:
>> So setzen SIE z.B. die (nur fuer endliche
>> Mengen zutreffende) Aussage implizit voraus, dass wenn zwischen
>> 2 Mengen A und B *eine* bijektive Abbildung existiert, auch *jede*
>> *andere* Abbildung zwischen den beiden Mengen bijektiv sein muesse
>
> Ich setze hier nichts dergleichen voraus. Ich setze lediglich voraus, dass es genau so viele natürliche Zahlen wie positive Ganzzahlbrüche gibt. Was ist dagegen einzuwenden? (Bevor Du antwortest versuche erst einmal das zu verstehen.)

"Genau so viele" ist bei *unendlichen* Mengen ein voellig sinnloser Begriff.
Das ist ja derade der Grund, weshalb man "Anzahl Elemente" bei Mengen durch
den Begriff der "MAechtigkeit" ersetzt hat, der bei endlichen Mengen zum
*selben* Ergenis fuehrt, aber im Gegensatz zu "Anzahl Elemente" auch fuer
unendliche Mengen sinnvoll ist.
"Anzahl Elemente" ist fuer unendliche Mengen deswegen ein sinnloser Begriff,
weil man mit dem IHNEN anscheinend vorschwebenden "Element fuer Element
nacheinander "durchzaehlen" bzw. "indizieren" niemals zu einem Ende kommen
wuerde und daher kein Ergebnis ermitteln koennte. Beim betrachten von (bi-
jektiven) Abbildungen ist das anders, da Abbildungen bekanntlich "Mengen"
und keine sequentiellen Prozesse sind.

>> SIE bemerken dabein gar nicht, an welchen Stellen
>> SIE unzutreffende Eigenschaften von unendlichen Mengen voraussetzen,
>> nur weil diese fuer *endliche* Mengen zutreffend sind.
>
> Wenn man die Elemente einer Menge sammelt und für jedes eingesammelte Element ein vorher eingesammeltes wieder wegwirft, so kann man die Sammlung nicht vervollständigen. Das gilt wohl für endliche wie für unendliche Mengen.

Voellig sinnloses Blabla.

>> Und um die
>> Inkonsistenzen zu uebertuenchen, erfinden SIE dann so seltsame nicht
>> existierende Dinge wie "potentiell unendliche Mengen" oder "dunkle
>> Zahlen" ...
>
> Welche Inkonsistenz meinst Du entdeckt zu haben?

<seufz/>

Tschuess,
Juergen Ilse (jue...@usenet-verwaltung.de)

Fritz Feldhase

unread,
May 17, 2022, 10:10:10 AM5/17/22
to
On Tuesday, May 17, 2022 at 3:55:36 PM UTC+2, Fritz Feldhase wrote:
> On Monday, May 16, 2022 at 7:13:40 PM UTC+2, Ganzhinterseher wrote
>
> WM betrachtet die Matrix M1:
>
> 1/1, 1/2, 1/3, ...
> 2/1, 2/2, 2/3, ...
> 3/1, 3/2, 3/3, ...
> 4/1, 4/2, 4/3, ...
> ...
>
> Diese ist wie folgt definiert:
>
> M1_n,m = n/m für alle n,m e IN.
>
> Ja, schön. Man kann zudem auch die Matrix M2 betrachten:
>
> 1/1, 1/2, 1/4 ...
> 1/3, 1/5, 1/8 ...
> 1/6, 1/9, 1/13 ...
> 1/10, 1/14, ...
> ...
>
> Diese ist wie folgt definiert:
>
> M2_n,m = 1/[(m + n - 1)(m + n - 2)/2 + m] für alle n,m e IN.

Man kann diese beiden Matrizen nun "gedanklich" zur Deckung bringen und "sieht" so, dass jeder Bruch der Form n/m genau einem Bruch der Form 1/k zugordnet ist (und umgekehrt). [Die "Zuordnung" ist über M1_n,m <-> M2_n,m für alle n,m e IN gegeben.]

Offenbar gibt es also zwischen der Menge {1/k : k e IN} und der Menge {n/m : n,m e IN} eine Bijektion. Im Kontext der Mengenlehre sagt man dann auch, dass die beiden Mengen gleichmächtig sind (vgl. Cantor).

Juergen Ilse

unread,
May 17, 2022, 10:10:56 AM5/17/22
to
Hallo,
Die Frage, was daran richtig ist, laessst sich leichter beantwworten: NICHTS

Tschuess,
Juergen Ilse (jue...@usenet-verwaltung.de)

Juergen Ilse

unread,
May 17, 2022, 10:13:57 AM5/17/22
to
Hallo,
Nein. Aber die Annahme, wenn es ei unendlichen Mengen *eine* bijektive
Abbildung zwischen zwei Mengen gibt, dann *jede* injektive Abbildung
auch surjektiv sein muesse, ist ein Fehlschluss.

Tschuess,
Juergen Ilse (jue...@usenet-verwaltung.de)

Gus Gassmann

unread,
May 17, 2022, 10:34:55 AM5/17/22
to
Vor allem die in diesem Kontext hirnrissig dämliche Verlust- und Entschädigungsrechnung. Also eigentlich alles. Please FUCK OFF.

Stefan Schmitz

unread,
May 17, 2022, 10:36:25 AM5/17/22
to
Stimmt nicht.
Die Brüche existieren alle. Und die Rechtschreibung scheint auch okay zu
sein.

Ganzhinterseher

unread,
May 17, 2022, 3:59:51 PM5/17/22
to
JVR schrieb am Dienstag, 17. Mai 2022 um 15:50:33 UTC+2:
> On Tuesday, May 17, 2022 at 2:48:42 PM UTC+2, Ganzhinterseher wrote:
> > JVR schrieb am Dienstag, 17. Mai 2022 um 14:41:09 UTC+2:
> >
> > > Ein Fehler (es bei weitem nicht der einzige) kommt daher, dass Sie den undefinierten
> > > Begriff "genau so viele" unkritisch verwenden.
> > Die Mengenlehre erfordert also dass man nicht genau so viele natürliche Zahlen wie Ganzzahlbrüche annehmen darf? Das sollte man den Erstsemestern freilich nicht mitteilen, sonst wären die meisten wohl gleichzeitig Letztsemester.

> Als Lehrer, wenn Sie einer wären

Das ist amtlich verifiziert und nicht von absichtlichen Fehleinschätzungen durch böswillige Laien wie Dich abhängig.

> (damit meine ich jemand, der 'Wissen vermittelt', nicht jemand wie Sie, der
> Abgeschriebenes repetiert)

Wo könnte ich denn die Erkenntnisse zu dunklen Zahlen abgeschrieben haben?

>, wären Sie also der Frage ausgesetzt, wie es denn sein kann, dass es
> halb so viele gerade Zahlen wie Ganze geben kann, wenn es doch offensichtlich (n <-> 2n) gleich viele sind.

Das Offensichtliche ist nicht immer das Zutreffende. Die sogenannte Bijektion ist keine. Die naive Behauptung, die Abbildung würde für alle natürlichen Zahlen gelten, ist leicht zu widerlegen: Jede natürliche Zahl, für die diese Abbildung geprüft werden kann, hat aleph_0 Nachfolger, für die die Abbildung also nicht geprüft werden kann.

Gruß, WM

Ganzhinterseher

unread,
May 17, 2022, 4:06:59 PM5/17/22
to
Juergen Ilse schrieb am Dienstag, 17. Mai 2022 um 16:07:46 UTC+2:
> Ganzhinterseher <askas...@gmail.com> wrote:
> > Juergen Ilse schrieb am Dienstag, 17. Mai 2022 um 05:31:12 UTC+2:
> >> So setzen SIE z.B. die (nur fuer endliche
> >> Mengen zutreffende) Aussage implizit voraus, dass wenn zwischen
> >> 2 Mengen A und B *eine* bijektive Abbildung existiert, auch *jede*
> >> *andere* Abbildung zwischen den beiden Mengen bijektiv sein muesse
> >
> > Ich setze hier nichts dergleichen voraus. Ich setze lediglich voraus, dass es genau so viele natürliche Zahlen wie positive Ganzzahlbrüche gibt. Was ist dagegen einzuwenden? (Bevor Du antwortest versuche erst einmal das zu verstehen.)
> "Genau so viele" ist bei *unendlichen* Mengen ein voellig sinnloser Begriff.

Nein, die Matheologie liefert völlig sinnlose Behauptungen.
Die Annahme, dass es genau so viele natürliche Zahlen wie natürliche Zahlen gibt, ist mathematisch gerechtfertigt. Die Annahme, dass es genau so viele natürliche Zahlen gibt wie positive Ganzzahlbrüche ebenfalls.

> Das ist ja derade der Grund, weshalb man "Anzahl Elemente" bei Mengen durch
> den Begriff der "MAechtigkeit" ersetzt hat, der bei endlichen Mengen zum
> *selben* Ergenis fuehrt, aber im Gegensatz zu "Anzahl Elemente" auch fuer
> unendliche Mengen sinnvoll ist.

Das ist eine falsche Behauptung.

> "Anzahl Elemente" ist fuer unendliche Mengen deswegen ein sinnloser Begriff,
> weil man mit dem IHNEN anscheinend vorschwebenden "Element fuer Element
> nacheinander "durchzaehlen" bzw. "indizieren" niemals zu einem Ende kommen
> wuerde und daher kein Ergebnis ermitteln koennte.

Das ist aber die Grundlage der Anzählung und des Mächtigkeitsbegriffs.

> Beim betrachten von (bi-
> jektiven) Abbildungen ist das anders, da Abbildungen bekanntlich "Mengen"
> und keine sequentiellen Prozesse sind.

Das ist falsch, lediglich eine Schutzbehauptung für Dummköpfe und Betrüger. Die Betrüger sind aber leicht zu entlarven: "Dabei nenne ich zwei wohlgeordnete Mengen von demselben Typus und schreibe ihnen gleiche Anzahl zu, wenn sie sich unter Wahrung der festgesetzten Rangordnung ihrer Elemente gegenseitig eindeutig aufeinander abbilden, oder wie man sich gewöhnlich ausdrückt, aufeinander abzählen lassen." [G. Cantor, letter to W. Wundt (5 Oct 1883)] Die Dummköpfe kriegen das leider nicht mitund wieder holen ihre Fehleinschätzung permanent.

> > Wenn man die Elemente einer Menge sammelt und für jedes eingesammelte Element ein vorher eingesammeltes wieder wegwirft, so kann man die Sammlung nicht vervollständigen. Das gilt wohl für endliche wie für unendliche Mengen.
> Voellig sinnloses Blabla.

Strenge Dich stärker an.

Gruß, WM

Ganzhinterseher

unread,
May 17, 2022, 4:12:36 PM5/17/22
to
Fritz Feldhase schrieb am Dienstag, 17. Mai 2022 um 16:10:10 UTC+2:
> On Tuesday, May 17, 2022 at 3:55:36 PM UTC+2, Fritz Feldhase wrote:
> > On Monday, May 16, 2022 at 7:13:40 PM UTC+2, Ganzhinterseher wrote
> >
> > WM betrachtet die Matrix M1:
> >
> > 1/1, 1/2, 1/3, ...
> > 2/1, 2/2, 2/3, ...
> > 3/1, 3/2, 3/3, ...
> > 4/1, 4/2, 4/3, ...
> > ...
> >
> > Diese ist wie folgt definiert:
> >
> > M1_n,m = n/m für alle n,m e IN.
> >
> > Ja, schön. Man kann zudem auch die Matrix M2 betrachten:
> >
> > 1/1, 1/2, 1/4 ...
> > 1/3, 1/5, 1/8 ...
> > 1/6, 1/9, 1/13 ...
> > 1/10, 1/14, ...
> > ...
> >
> > Diese ist wie folgt definiert:
> >
> > M2_n,m = 1/[(m + n - 1)(m + n - 2)/2 + m] für alle n,m e IN.
> Man kann diese beiden Matrizen nun "gedanklich" zur Deckung bringen und "sieht" so, dass jeder Bruch der Form n/m genau einem Bruch der Form 1/k zugordnet ist (und umgekehrt). [Die "Zuordnung" ist über M1_n,m <-> M2_n,m für alle n,m e IN gegeben.]
>
Selbst der Seher, der dies sieht, kann nicht verleugnen, dass die Indizierung der Matrix M1 nicht vollständig möglich ist, weil für jeden indizierten Bruch ein anderer Bruch derselben Matrix diesen Index verliert. Allerdings werden alle Brüche der Matrix M2 indiziert.

Wodurch wird dieses Dilemma gelöst? Durch Verschweigen des einen Sachverhaltes?

Gruß, WM

Ganzhinterseher

unread,
May 17, 2022, 4:20:13 PM5/17/22
to
Juergen Ilse schrieb am Dienstag, 17. Mai 2022 um 16:13:57 UTC+2:
> Ganzhinterseher <askas...@gmail.com> wrote:

> >> > 1/1, 1/2, 1/3, 1/4, ...
> >> > 2/1, 2/2, 2/3, 2/4, ...
> >> > 3/1, 3/2, 3/3, 3/4, ...
> >> > 4/1, 4/2, 4/3, 4/4, ...
> >> > 5/1, 5/2, 5/3, 5/4, ...
> >> ??
> >
> > Die Bijektion der natürlichen Zahlen mit den Ganzzahlbrüchen ist eine Fehlüberlegung?
> Nein. Aber die Annahme, wenn es ei unendlichen Mengen *eine* bijektive
> Abbildung zwischen zwei Mengen gibt, dann *jede* injektive Abbildung
> auch surjektiv sein muesse, ist ein Fehlschluss.

Hier wird diese Tatsache ja nicht angewandt, um die Matheologen nicht zu verschrecken. Es wird lediglich die Probe gemacht, ob alle Indizes vorhanden sind. Zu dem Zweck werden sie mit den Ganzzahlbrüchen der ersten Spalte in Bijektion gesetzt. Die auf diese Weise zertifizierten natürlichen Zahlen werden nun als Indizes nach Cantors Formel verteilt. Sie indizieren die Brüche m/n, wie von Cantor angegeben
k = (m + n - 1)(m + n - 2)/2 + m
mit dem Ergebnis
1/1, 1/2, 2/1, 1/3, 2/2, 3/1, 1/4, 2/3, 3/2, 4/1, 1/5, ... .

Allerdings steht fest dass sich der Anteil der nicht indizierten Brüche in der Matrix nicht verändern kann, weil jeder neu vergebene Index von einem vorher schon indizierten Bruch fortgenommen wird. Das ist unbestreitbar.

Gruß, WM

Ganzhinterseher

unread,
May 17, 2022, 4:23:53 PM5/17/22
to
Gus Gassmann schrieb am Dienstag, 17. Mai 2022 um 16:34:55 UTC+2:

> > Was findest Du denn falsch?
> Vor allem die in diesem Kontext hirnrissig dämliche Verlust- und Entschädigungsrechnung.

Ja, sie ist der Sargnagel der Matheologie. Aber was hast Du sonst daran auszusetzen?

Zunächst wird lediglich die Probe gemacht, ob alle Indizes vorhanden sind. Zu diesem Zweck werden sie mit den Ganzzahlbrüchen der ersten Spalte in Bijektion gesetzt. Die auf diese Weise auf Vollzähligkeit geprüften natürlichen Zahlen werden nun als Indizes nach Cantors Formel verteilt. Sie indizieren die Brüche m/n, wie von Cantor angegeben
k = (m + n - 1)(m + n - 2)/2 + m
mit dem Ergebnis
1/1, 1/2, 2/1, 1/3, 2/2, 3/1, 1/4, 2/3, 3/2, 4/1, 1/5, ... .

Allerdings steht fest dass sich der Anteil der nicht indizierten Brüche in der Matrix nicht verändern kann, weil jeder neu vergebene Index von einem vorher schon indizierten Bruch fortgenommen wird. Was wäre daran nicht exakt mathematisch nachprüfbar?

Gruß, WM

Ganzhinterseher

unread,
May 17, 2022, 4:31:24 PM5/17/22
to
JVR schrieb am Dienstag, 17. Mai 2022 um 14:41:09 UTC+2:
>
> Ein Fehler (es bei weitem nicht der einzige) kommt daher, dass Sie den undefinierten
> Begriff "genau so viele" unkritisch verwenden.

Vom Standpunkt eines Matheologen betrachtet, muss es ja geradezu pervers erscheinen, die Anzahl der natürlichen Zahlen durch die Anzahl der positiven Ganzzahlbrüche darstellen zu lassen.

Gruß, WM

Rainer Rosenthal

unread,
May 17, 2022, 4:37:32 PM5/17/22
to
Am 17.05.2022 um 22:06 schrieb Ganzhinterseher:
> Die Betrüger sind aber leicht zu entlarven.

Nein, es kostet schon etwas Zeit und Nerven, sie zu entlarven.
Dass das Entlarven geklappt hat, stellt man daran fest, dass sie
plötzlich stumm werden und nicht mehr wissen, wie sie den Quatsch auf
Seite 37 verteidigen sollen.

Gruß,
RR

JVR

unread,
May 17, 2022, 5:00:10 PM5/17/22
to
Mücke, Sie sind langweilig.

Rainer Rosenthal

unread,
May 18, 2022, 3:53:20 AM5/18/22
to
Am 17.05.2022 um 22:12 schrieb Ganzhinterseher:
> Selbst der Seher, der dies sieht, kann nicht verleugnen, dass ...

Aber Du Schwätzer, der hier schwatzt, verleugnest, dass Du Sätze kürzen
musst, um länger mitreden zu können.

Gruß,
RR


Gus Gassmann

unread,
May 18, 2022, 6:25:23 AM5/18/22
to
On Tuesday, 17 May 2022 at 17:23:53 UTC-3, Ganzhinterseher wrote:
[...]
> Allerdings steht fest dass sich der Anteil der nicht indizierten Brüche in der Matrix nicht verändern kann,

Das ist doch alles kalter Kaffee und hat mit dem eigentlichen Problem aber auch überhaupt nichts zu tun, nämlich, dass du keine Ahnung hast worüber du schwätzt. Bei jedem endlich indizierten Schritt sind aleph_0 Brüche noch nicht abgearbeitet, und es bleiben aleph_0 Ganzzahlbrüche in der ersten Spalte, um sie zu indizieren. Dass ein unendlicher schrittweiser Prozess in endlich vielen Schritten noch nicht zu Ende ist, ist das deine weltbewegende Erkenntnis?!?
Message has been deleted

Fritz Feldhase

unread,
May 18, 2022, 6:59:15 AM5/18/22
to
On Tuesday, May 17, 2022 at 10:23:53 PM UTC+2, Ganzhinterseher wrote:

> Allerdings steht fest dass sich der Anteil der nicht indizierten Brüche in der Matrix nicht verändern kann

In der Tat. Dieser Anteil ist übrigens 0.

Beweis: Die von Cantor angegebene Folge (1/1, 1/2, 2/1, 1/3, 2/2, 3/1, 1/4, 2/3, 3/2, 4/1, 1/5, ...) enthält ALLE Brüche als Terme (und jeden nur einmal).

Sie verstehen: Die Folge enthält alle Brüche (und jeden nur einmal) und die Matrix enthält alle Brüche (und jeden nur einmal). Da die Folge alle Brüche enthält (und jeden nur einmal) sind alle Brüche indiziert, also auch alle Brüche in der Matrix.

Ganzhinterseher

unread,
May 18, 2022, 9:58:28 AM5/18/22
to
Gus Gassmann schrieb am Mittwoch, 18. Mai 2022 um 12:25:23 UTC+2:
> Bei jedem endlich indizierten Schritt sind aleph_0 Brüche noch nicht abgearbeitet, und es bleiben aleph_0 Ganzzahlbrüche in der ersten Spalte, um sie zu indizieren.

Richtig, nur ist aleph_0 keine quantitative Angabe. Es verbleiben im n-ten Schritt |N| - n Ganzzahlbrüche in der ersten Spalte, es sind aber in jedem Schritt |N|^2 - |N| zu indizieren.

> Dass ein unendlicher schrittweiser Prozess in endlich vielen Schritten noch nicht zu Ende ist, ist das deine weltbewegende Erkenntnis?!?

Dass ein unendlicher schrittweiser Prozess nach allen unendlich vielen Schritten nicht fertig ist, weil |N|^2 - |N| Aufgaben auch nach unendlich vielen Schritten unerledigt sind, aber kein Schritt mehr möglich ist, sie zu erledigen, das ist meine Erkenntnis. Diese Erkenntnis ist zwar nicht neu, aber ich haben sie noch nie so klar und einprägsam modellieren können.

Gruß, WM

Ganzhinterseher

unread,
May 18, 2022, 10:05:36 AM5/18/22
to
Fritz Feldhase schrieb am Mittwoch, 18. Mai 2022 um 12:59:15 UTC+2:
> On Tuesday, May 17, 2022 at 10:23:53 PM UTC+2, Ganzhinterseher wrote:
>
> > Allerdings steht fest dass sich der Anteil der nicht indizierten Brüche in der Matrix nicht verändern kann
> In der Tat. Dieser Anteil ist übrigens 0.

Rechenfehler. Der Anteil ist am Anfang |N|^2 - |N| und bleibt in allen unendlich vielen Schritten, ob sie nun schrittweise oder alle gleichzeitig erledigt werden, konstant.
>
> Beweis: Die von Cantor angegebene Folge (1/1, 1/2, 2/1, 1/3, 2/2, 3/1, 1/4, 2/3, 3/2, 4/1, 1/5, ...) enthält ALLE Brüche als Terme (und jeden nur einmal).

So glaubten bisher viele.
>
> Sie verstehen: Die Folge enthält alle Brüche

Sie enthält alle Brüche, auf die noch unendlich viele folgen, doch diese unendlich vielen enthält sie nicht, denn dann würden sie ja nicht mehr folgen.

> (und jeden nur einmal) und die Matrix enthält alle Brüche (und jeden nur einmal). Da die Folge alle Brüche enthält (und jeden nur einmal) sind alle Brüche indiziert, also auch alle Brüche in der Matrix.

Selbst wenn ich Deinen Fehler nicht finden und angeben könnte, hätten wir mit meinem Modell einen Widerspruch. Leider bist Du weder in der Lage ihn zu erkennen, noch mein Modell in Dein Bewusstsein vordringen zu lassen. Das ist der beste Beweis für die matheologische Barrierebildung

Gruß, WM

Gus Gassmann

unread,
May 18, 2022, 10:49:51 AM5/18/22
to
"klar und einprägsam" = "stumpfsinnig"

Gus Gassmann

unread,
May 18, 2022, 10:51:45 AM5/18/22
to
On Wednesday, 18 May 2022 at 11:05:36 UTC-3, Ganzhinterseher wrote:
> Fritz Feldhase schrieb am Mittwoch, 18. Mai 2022 um 12:59:15 UTC+2:
> > On Tuesday, May 17, 2022 at 10:23:53 PM UTC+2, Ganzhinterseher wrote:
> >
> > > Allerdings steht fest dass sich der Anteil der nicht indizierten Brüche in der Matrix nicht verändern kann
> > In der Tat. Dieser Anteil ist übrigens 0.
> Rechenfehler. Der Anteil ist am Anfang |N|^2 - |N|

Es wird allerdings immer trauriger. Jetzt behauptet der gute Herr Professor also schon |N|^2 - |N| sei ein "Anteil". Hirnrissiger geht's nicht.

Fritz Feldhase

unread,
May 18, 2022, 11:01:03 AM5/18/22
to
On Wednesday, May 18, 2022 at 4:05:36 PM UTC+2, Ganzhinterseher wrote:
> Fritz Feldhase schrieb am Mittwoch, 18. Mai 2022 um 12:59:15 UTC+2:
> >
> > Die von Cantor angegebene Folge (1/1, 1/2, 2/1, 1/3, 2/2, 3/1, 1/4, 2/3, 3/2, 4/1, 1/5, ...) enthält ALLE Brüche als Terme (und jeden nur einmal).
> >
> So glaubten bisher viele.

In der Tat.

> > Sie verstehen: Die Folge enthält alle Brüche
> >
> Sie enthält alle Brüche, auf die noch unendlich viele folgen, doch diese unendlich vielen enthält sie nicht, denn dann würden sie ja nicht mehr folgen.

Das ist ein bestechendes Argument! Demnach entält auch die Folge aller natürlichen Zahlen (1, 2, 3, ...) nicht alle natürlichen Zahlen?

Denn Ihr Argument, auf diesen Fall angewendet, laute ja: "Sie enthält alle Zahlen auf die noch unendlich viele folgen, doch diese unendlich vielen enthält sie nicht, denn dann würden sie ja nicht mehr folgen."

Ja, Mückenheim, das ist wirklich genial. Mit diesem Argument haben Sie - in einem einzigen Satz - die Existenz der (unendlichen) Menge der natürlichen Zahlen widerlegt. Damit ist der sog. transfiniten Mengenlehre die Basis entzogen und sie als der Unsinn nachgewiesen, der sie ist!

Um es kurz zusammenzufassen: Es gibt keine (aktual) unendlichen Folgen (je paarweise verschiedener Terme). Insbesondere weder die Folge aller natürlichen Zahlen (in natürlicher Reihenfolge), noch die von Cantor angegebenen Folge, von der behauptet wurde, dass sie alle Brüche (als Terme) enthalten würde. - Man kann diese beiden Sachverhalte auch nicht getrennt betrachten, da sie b e i d e durch ein und dasselbe geniale Argument "erledigt" werden.

Damit ist (eigentlich) alles gesagt.

Nur eine Frage hätte ich noch. In Ihrem bestechenden Argument sagen Sie "doch diese unendlich vielen enthält sie nicht". Nun frage ich mich, worauf - also auf welche Zahlen - sich dieses/Ihr "diese" bezieht.

Sie verstehen: Auf die Zahl 1 folgen die unendlich vielen Zahlen 2, 3, 4, 5, ... Nun kann sich Ihr "diese" aber NICHT auf all DIESE Zahlen beziehen, denn bekanntlich enthält die Folge der natürlichen Zahlen auch (insbesondere) die Zahlen 2, 3, 4, 5. Wenn wir nun aber z. B. den Abschnitt 1, 2, 3, 4, 5 der Folge betrachten, dann folgen auf 5 die Zahlen 6, 7, 8, 9, ... Aber Ihr "diese" kann sich auch NICHT auf all DIESE Zahlen beziehen, denn bekanntlich enthält die Folge der natürlichen Zahlen neben 1, 2, 3, 4, 5 auch die Zahlen 6, 7, 8, 9, usw. Generell gilt doch, dass die Folge der natürlichen Zahlen mit n auch n+1 (als Term) enthält. Demnach scheint sich Ihr "diese" auf gar keine Zahlen beziehen zu können. Sie verstehen: 1 ist sicher nicht eine "dieser unendlich vielen Zahlen" und wenn n keine "dieser unendlich vielen Zahlen" ist, dann ist es auch n+1 nicht. Demzufolge ist aber KEINE natürliche Zahle eine "dieser unendlich vielen Zahlen" (das folgt mithilfe der sog. Induktion).

Ihr Argument scheint mir hier einen entscheidenden (sic!) Fehler zu enthalten.

Sie verstehen: Wenn 3 Männer vor dem Richter stehen und sein Urteil lautet, d i e s e 3 Männer hinrichten zu lassen, so wird dem Urteil nicht entsprochen, wenn man stattdessen 3 a n d e r e Männer hinrichtet.

Ralf Bader

unread,
May 18, 2022, 12:29:08 PM5/18/22
to
On 05/18/2022 12:59 PM, Fritz Feldhase wrote:
> On Tuesday, May 17, 2022 at 10:23:53 PM UTC+2, Ganzhinterseher wrote:
>
>> Allerdings steht fest dass sich der Anteil der nicht indizierten Brüche in der Matrix nicht verändern kann
>
> In der Tat. Dieser Anteil ist übrigens 0.
>
> Beweis: Die von Cantor angegebene Folge (1/1, 1/2, 2/1, 1/3, 2/2, 3/1, 1/4, 2/3, 3/2, 4/1, 1/5, ...) enthält ALLE Brüche als Terme (und jeden nur einmal).
>
> Sie verstehen:

Nein, er versteht nicht.

Ralf Bader

unread,
May 18, 2022, 12:32:44 PM5/18/22
to
On 05/18/2022 04:05 PM, Ganzhinterseher wrote:
> Fritz Feldhase schrieb am Mittwoch, 18. Mai 2022 um 12:59:15 UTC+2:
>> On Tuesday, May 17, 2022 at 10:23:53 PM UTC+2, Ganzhinterseher
>> wrote:
>>
>>> Allerdings steht fest dass sich der Anteil der nicht indizierten
>>> Brüche in der Matrix nicht verändern kann
>> In der Tat. Dieser Anteil ist übrigens 0.
>
> Rechenfehler. Der Anteil ist am Anfang |N|^2 - |N| und bleibt in
> allen unendlich vielen Schritten, ob sie nun schrittweise oder alle
> gleichzeitig erledigt werden, konstant.
>>
>> Beweis: Die von Cantor angegebene Folge (1/1, 1/2, 2/1, 1/3, 2/2,
>> 3/1, 1/4, 2/3, 3/2, 4/1, 1/5, ...) enthält ALLE Brüche als Terme
>> (und jeden nur einmal).
>
> So glaubten bisher viele.

Mückenheim, Sie sind für Mathematik zu doof und zu blöde.

Ralf Bader

unread,
May 18, 2022, 12:34:03 PM5/18/22
to
Nein, er versteht nicht.

Fritz Feldhase

unread,
May 18, 2022, 12:41:53 PM5/18/22
to
Das ist mir schon klar.

Gemeint ist hier immer: "Auch wenn Sie es nicht verstehen:"

Im Grund ist es mir scheißegal, ob e r versteht oder nicht. Ich will das, was nach den ":" kommt, lediglich (mal) gesagt haben.

Tom Bola

unread,
May 18, 2022, 1:43:49 PM5/18/22
to
Fritz Feldhase schrieb:

> On Wednesday, May 18, 2022 at 6:34:03 PM UTC+2, Ralf Bader wrote:
>> On 05/18/2022 05:01 PM, Fritz Feldhase wrote:
>>>
>>> Sie verstehen:
>>>
>> Nein, er versteht nicht.
>
> Das ist mir schon klar.

Dann hör doch endlich wieder auf...

Fritz Feldhase

unread,
May 18, 2022, 2:32:08 PM5/18/22
to
Tom Bola rührt die Tombola!

Fritz Feldhase

unread,
May 18, 2022, 3:16:13 PM5/18/22
to
On Wednesday, May 18, 2022 at 5:01:03 PM UTC+2, Fritz Feldhase wrote:
> On Wednesday, May 18, 2022 at 4:05:36 PM UTC+2, Ganzhinterseher wrote:
> > >
> > > Die Folge enthält alle Brüche ... [FF]
> > >
> > Sie enthält alle Brüche, auf die noch unendlich viele folgen, doch diese unendlich vielen enthält sie nicht, denn dann würden sie ja nicht mehr folgen. [GH]
> >
> Das ist ein bestechendes Argument! [FF]

[Jedoch...]

Bei reiflicher Überlegung will es mir scheinen, dass Sie hier wieder die bekanntermaßen unzulässige "Schlussweise" der Quantorvertauschung angewendet haben.

JA: "Für jeden Bruch der Folge existieren unendlich viele Brüche (in der Folge), die auf ihn folgen."

NEIN: "Es existieren unendlich viele Brüche (in der Folge), die auf jeden Bruch der Folge folgen."

Das ist eine Variante des Fehlschlusses:

Ax Ey ... => Ey Ax ...

Siehe: https://en.wikipedia.org/wiki/Quantifier_shift

Die Verwendung des Wortes "diese" in

"... doch diese unendlich vielen enthält sie nicht, denn ..."

liefert einen klaren Hinweis darauf, dass Sie hier wieder einmal in die Irre (sic!) gegangen sind.

Hinweis: Für jedes Glied a in der Folge der natürlichen Zahlen gibt es ein Glied b (in der Folge), das größer ist als a. Es gibt aber kein Glied b (in der Folge), das größer ist als alle Glieder a in der Folge. Daraus kann man nun NICHT schließen, dass es natürliche Zahlen gibt, die nicht Glieder der Folge aller natürlicher Zahlen (in natürlicher Ordnung) sind. Naja, jedenfalls nicht im Rahmen der Mathematik. In Ihrem Wahnsystem kann man das natürlich schon, Mückenheim, oder? Entweder das, oder es beweist die Existenz der braunen Kackzahlen.

Juergen Ilse

unread,
May 18, 2022, 4:33:29 PM5/18/22
to
Hallo,

Ganzhinterseher <askas...@gmail.com> wrote:
> Juergen Ilse schrieb am Dienstag, 17. Mai 2022 um 16:07:46 UTC+2:
>> "Anzahl Elemente" ist fuer unendliche Mengen deswegen ein sinnloser Begriff,
>> weil man mit dem IHNEN anscheinend vorschwebenden "Element fuer Element
>> nacheinander "durchzaehlen" bzw. "indizieren" niemals zu einem Ende kommen
>> wuerde und daher kein Ergebnis ermitteln koennte.
>
> Das ist aber die Grundlage der Anzählung und des Mächtigkeitsbegriffs.

Nein. Die Grundlage des MAechtigkeitsbegriffs sind Abbildungen.

>> Beim betrachten von (bi-
>> jektiven) Abbildungen ist das anders, da Abbildungen bekanntlich "Mengen"
>> und keine sequentiellen Prozesse sind.
>
> Das ist falsch,

Was soll daran falsch sein? Was ist eine Abbildung, wenn nicht eine Teilmenge
des kartesischen Produkts zweier Mengen?

> lediglich eine Schutzbehauptung für Dummköpfe und Betrüger. Die Betrüger sind aber leicht zu entlarven: "Dabei nenne ich zwei wohlgeordnete Mengen von demselben Typus und schreibe ihnen gleiche Anzahl zu, wenn sie sich unter Wahrung der festgesetzten Rangordnung ihrer Elemente gegenseitig eindeutig aufeinander abbilden, oder wie man sich gewöhnlich ausdrückt, aufeinander abzählen lassen." [G. Cantor, letter to W. Wundt (5 Oct 1883)]

wenn SIE Cantors Texte woertlich nehmen wollen, sollten SIE sich dazu auch
der Cantorschen Bedeutung der Begriffe bedienen (die sich von der umgangs-
sprachlichen Bedeutung ebenso wie von der Bedeutung in der moernen Mathematik.

Im uebrigen bezieht sich die zitierte Textstelle nicht auf Gleichmaechtigkeit
(was man auch daran erkennt, dass hier nicht von beliebigen Mengen sondern von
Wohlordnungen die Rede ist). Gleichmaechtigkeiit setzt keine Ordnung auf den
beteiligten Mengen voraus.

Tschuess,
Juergen Ilse (jue...@usenet-verwaltung.de)

Fritz Feldhase

unread,
May 18, 2022, 4:45:07 PM5/18/22
to
On Wednesday, May 18, 2022 at 10:33:29 PM UTC+2, Juergen Ilse wrote:
> Hallo,
>
> Ganzhinterseher <askas...@gmail.com> wrote:
> > Juergen Ilse schrieb am Dienstag, 17. Mai 2022 um 16:07:46 UTC+2:
> > >
> > > da Abbildungen bekanntlich "Mengen" und keine sequentiellen Prozesse sind.
> > >
> > Das ist falsch,
> >
> Was soll daran falsch sein?

Die Frage ist falsch gestellt.

Es ist falsch, _weil_ der GRÖMAZ der Meinung ist, dass es falsch ist.

D. h. es ist ausgeschlossen, dass es richtig ist!

So einfach ist das.

Du erinnerst Dich doch hoffentlich: "Everyone in set theory is wrong." (W. Mückenheim, sci.math)

[Except, W. Mückeheim that is.]

Juergen Ilse

unread,
May 18, 2022, 5:18:24 PM5/18/22
to
Hallo,
Ein "unendlicher schrittweiser Prozess" ist *niemals* beendet. Das ist der
Grund, weshalb man Abbildungen unendlicher Mengen eben *nicht* als Prozess
sieht, sondern als eine Teilmenge des kartesischen Produkts von Definitions-
menge und Bildmenge.

WM ist offenbvar nicnht in der Lage, zu begreifen, dass Abbildungen unend-
licher Menge *niemals* als "Prozess" angesehen werden koennen und eben etwas
*voellig* *anderes* sind.

Tschuess,
Juergen Ilse (jue...@usenet-verwaltung.de)

Juergen Ilse

unread,
May 18, 2022, 5:20:45 PM5/18/22
to
Ganzhinterseher <askas...@gmail.com> wrote:
> Gus Gassmann schrieb am Mittwoch, 18. Mai 2022 um 12:25:23 UTC+2:
>> Bei jedem endlich indizierten Schritt sind aleph_0 Brüche noch nicht abgearbeitet, und es bleiben aleph_0 Ganzzahlbrüche in der ersten Spalte, um sie zu indizieren.
>
> Richtig, nur ist aleph_0 keine quantitative Angabe.

Selbstverstaendlich ist aleph0 eine "quantitative Angabe" wie jede andere
Kardinalzahl auch.

Tschuess,
Juergen Ilse (jue...@usenet-verwaltung.de)

Ganzhinterseher

unread,
May 19, 2022, 7:50:17 AM5/19/22
to
Fritz Feldhase schrieb am Mittwoch, 18. Mai 2022 um 17:01:03 UTC+2:
> On Wednesday, May 18, 2022 at 4:05:36 PM UTC+2, Ganzhinterseher wrote:

> > Sie enthält alle Brüche, auf die noch unendlich viele folgen, doch diese unendlich vielen enthält sie nicht, denn dann würden sie ja nicht mehr folgen.
> Das ist ein bestechendes Argument! Demnach entält auch die Folge aller natürlichen Zahlen (1, 2, 3, ...) nicht alle natürlichen Zahlen?

Wenn es nach Dir ginge, dann nicht.
Es gibt ℵo Nachfolger zwischen jeder natürlichen Zahl n und ω.
Also existiert keine natürliche Zahl mit weniger Nachfolgern zwischen ihr und ω.
Das impliziert, dass zwischen allen Elementen der Menge der natürlichen Zahlen und ω ℵo Nachfolger existieren.
Das impliziert, dass zwischen der Menge der natürlichen Zahlen und ω ℵo Nachfolger existieren.

> Denn Ihr Argument, auf diesen Fall angewendet, laute ja: "Sie enthält alle Zahlen auf die noch unendlich viele folgen, doch diese unendlich vielen enthält sie nicht, denn dann würden sie ja nicht mehr folgen."

Die Lösung ist einfach, aber ungewohnt: Alle definierbaren natürlichen Zahlen bilden eine potentiell unendliche Kollektion. Darauf folgen fast alle natürlichen Zahlen. Sie sind dunkel.

Um die Lösung einleuchtender zu erklären, benutze ich die Abzählung der Brüche. Da sich dabei die Anteile der abgezählten und der nicht abgezählten Brüche niemals ändern, aber alle definierbaren Brüche mit definierbaren Indizes nach Cantor abgezählt werden, ist die Existenz der dunklen Brüche evident. Wer das einmal verstanden hat, der versteht auch, dass schon die Menge der natürlichen Zahlen einen dunklen Teil hat.
>
> Ja, Mückenheim, das ist wirklich genial. Mit diesem Argument haben Sie - in einem einzigen Satz - die Existenz der (unendlichen) Menge der natürlichen Zahlen widerlegt.

Keineswegs. Wir müssen nur den sorglosen Gebrauch des Wörtchens "unendlich" genauer fassen. Die definierbaren Zahlen, z.B. nach Peano, bilden eine potentiell unendliche Kollektion. Falls es aber eine aktual unendliche Menge gibt, dann existiert zwischen der Kollektion und omega eine aktual unendliche Menge dunkler natürlicher Zahlen, die u.a. das Reservoir für die Erschaffung weiterer definierter Zahlen bilden.

> Damit ist der sog. transfiniten Mengenlehre die Basis entzogen und sie als der Unsinn nachgewiesen, der sie ist!

Im Gegenteil.
>
> Um es kurz zusammenzufassen: Es gibt keine (aktual) unendlichen Folgen (je paarweise verschiedener Terme). Insbesondere weder die Folge aller natürlichen Zahlen (in natürlicher Reihenfolge), noch die von Cantor angegebenen Folge, von der behauptet wurde, dass sie alle Brüche (als Terme) enthalten würde. - Man kann diese beiden Sachverhalte auch nicht getrennt betrachten, da sie b e i d e durch ein und dasselbe geniale Argument "erledigt" werden.

Diese Folgen gibt es tatsächlich nicht. Es sind nur potentiell unendliche Folgen definierbar.
>
> Damit ist (eigentlich) alles gesagt.
>
> Nur eine Frage hätte ich noch. In Ihrem bestechenden Argument sagen Sie "doch diese unendlich vielen enthält sie nicht". Nun frage ich mich, worauf - also auf welche Zahlen - sich dieses/Ihr "diese" bezieht.

Das bezieht sich auf die dunklen Zahlen bzw. Brüche, von denen noch beliebig viele definierbar sind, jedoch immer aktual unendlich viele dunkel bleiben.
>
> Sie verstehen: Auf die Zahl 1 folgen die unendlich vielen Zahlen 2, 3, 4, 5, ... Nun kann sich Ihr "diese" aber NICHT auf all DIESE Zahlen beziehen, denn bekanntlich enthält die Folge der natürlichen Zahlen auch (insbesondere) die Zahlen 2, 3, 4, 5. Wenn wir nun aber z. B. den Abschnitt 1, 2, 3, 4, 5 der Folge betrachten, dann folgen auf 5 die Zahlen 6, 7, 8, 9, ... Aber Ihr "diese" kann sich auch NICHT auf all DIESE Zahlen beziehen, denn bekanntlich enthält die Folge der natürlichen Zahlen neben 1, 2, 3, 4, 5 auch die Zahlen 6, 7, 8, 9, usw. Generell gilt doch, dass die Folge der natürlichen Zahlen mit n auch n+1 (als Term) enthält. Demnach scheint sich Ihr "diese" auf gar keine Zahlen beziehen zu können. Sie verstehen: 1 ist sicher nicht eine "dieser unendlich vielen Zahlen" und wenn n keine "dieser unendlich vielen Zahlen" ist, dann ist es auch n+1 nicht. Demzufolge ist aber KEINE natürliche Zahle eine "dieser unendlich vielen Zahlen" (das folgt mithilfe der sog. Induktion).

Alle per Induktion erreichbaren oder definierbaren Zahlen gehören zu der potentiell unendlichen Kollektion, die man bisher für die gesamte Menge |N gehalten hat. Aber man vergaß, dass per Induktion auch gezeigt werden kann, dass jede Zahl dieser Kollektion aktual unendlich viele Nachfolger hat (wenn es aktual unendlich viele Zahlen gibt). Und damit haben alle Zahlen dieser Kollektion aktual unendlich viele Nachfolger. Denn es gibt keine definierbare Zahl, die näher an omega dran ist. Das zeigt sich auch, wenn man von omega herunterzählen will. Mann muss die nächste Zahl definieren. Und damit hat man bereits eine aktual unendliche Menge übersprungen. Man kann unendlich hinaufzählen, weil man zu jeder definierbaren Zahl den Nachfolger, falls er noch nicht existierte, erschaffen kann. Herunterwärts funktioniert das nicht.

Gruß, WM

Ganzhinterseher

unread,
May 19, 2022, 7:57:12 AM5/19/22
to
Fritz Feldhase schrieb am Mittwoch, 18. Mai 2022 um 21:16:13 UTC+2:

> Hinweis: Für jedes Glied a in der Folge der natürlichen Zahlen gibt es ein Glied b (in der Folge), das größer ist als a. Es gibt aber kein Glied b (in der Folge), das größer ist als alle Glieder a in der Folge. Daraus kann man nun NICHT schließen, dass es natürliche Zahlen gibt, die nicht Glieder der Folge aller natürlicher Zahlen (in natürlicher Ordnung) sind.

Die Quantorenvertauschung ist hier offensichtlich falsch. Sie ist aber nicht immer falsch. Im folgenden Beispiel ist sie offensichtlich richtig:

Es gibt ℵo Nachfolger zwischen jeder definierbaren natürlichen Zahl n und ω.
Also existiert keine def. natürliche Zahl mit weniger Nachfolgern zwischen ihr und ω.
Das impliziert, dass zwischen allen definierbaren Elementen der Menge der natürlichen Zahlen und ω ℵo Nachfolger existieren. Sonst müsste es nämlich mindestens ein def. Element mit weniger Nachfolgern geben.
Das impliziert, dass zwischen der Menge der def. natürlichen Zahlen und ω ℵo Nachfolger existieren.

Gruß, WM

Ganzhinterseher

unread,
May 19, 2022, 8:05:45 AM5/19/22
to
Juergen Ilse schrieb am Mittwoch, 18. Mai 2022 um 22:33:29 UTC+2:
> Hallo,
>
> Ganzhinterseher <askas...@gmail.com> wrote:
> > Juergen Ilse schrieb am Dienstag, 17. Mai 2022 um 16:07:46 UTC+2:
> >> "Anzahl Elemente" ist fuer unendliche Mengen deswegen ein sinnloser Begriff,
> >> weil man mit dem IHNEN anscheinend vorschwebenden "Element fuer Element
> >> nacheinander "durchzaehlen" bzw. "indizieren" niemals zu einem Ende kommen
> >> wuerde und daher kein Ergebnis ermitteln koennte.
> >
> > Das ist aber die Grundlage der Abzählung und des Mächtigkeitsbegriffs.
> Nein. Die Grundlage des MAechtigkeitsbegriffs sind Abbildungen.

Man kann die Abzählung abzählbarer Mengen auch als Abbildung bezeichnen. Das ändert aber nichts daran, dass eine Abzählung bis zu jedem n nachprüfbar existieren muss.

> >> Beim betrachten von (bi-
> >> jektiven) Abbildungen ist das anders, da Abbildungen bekanntlich "Mengen"
> >> und keine sequentiellen Prozesse sind.
> >
> > Das ist falsch,
> Was soll daran falsch sein? Was ist eine Abbildung, wenn nicht eine Teilmenge
> des kartesischen Produkts zweier Mengen?

Im Falle abz. Mengen ist es eine Abzählung.

> > lediglich eine Schutzbehauptung für Dummköpfe und Betrüger. Die Betrüger sind aber leicht zu entlarven: "Dabei nenne ich zwei wohlgeordnete Mengen von demselben Typus und schreibe ihnen gleiche Anzahl zu, wenn sie sich unter Wahrung der festgesetzten Rangordnung ihrer Elemente gegenseitig eindeutig aufeinander abbilden, oder wie man sich gewöhnlich ausdrückt, aufeinander abzählen lassen." [G. Cantor, letter to W. Wundt (5 Oct 1883)]

> wenn SIE Cantors Texte woertlich nehmen wollen, sollten SIE sich dazu auch
> der Cantorschen Bedeutung der Begriffe bedienen

Genau das tue ich.

> Im uebrigen bezieht sich die zitierte Textstelle nicht auf Gleichmaechtigkeit
> (was man auch daran erkennt, dass hier nicht von beliebigen Mengen sondern von
> Wohlordnungen die Rede ist). Gleichmaechtigkeiit setzt keine Ordnung auf den
> beteiligten Mengen voraus.

Das ist Larifari. Die Prüfung der Gleichmächtigkeit verschiedener Mengen setzt eine Ordnung voraus.

"Wenn zwei wohldefinierte Mannigfaltigkeiten M und N sich eindeutig und vollständig, Element für Element, einander zuordnen lassen (was, wenn es auf eine Art möglich ist, immer auch noch auf viele andere Weisen geschehen kann), so möge für das Folgende die Ausdrucksweise gestattet sein, daß diese Mannigfaltigkeiten gleiche Mächtigkeit haben, oder auch, daß sie äquivalent sind." [Cantor, p. 119]

Element für Element.

"Jeder wohldefinierten Menge kommt danach eine bestimmte Mächtigkeit zu, wobei zwei Mengen dieselbe Mächtigkeit zugeschrieben wird, wenn sie sich gegenseitig eindeutig, Element für Element einander zuordnen lassen." [Cantor, p. 167]

"Dabei nenne ich zwei wohlgeordnete Mengen von demselben Typus und schreibe ihnen gleiche Anzahl zu, wenn sie sich unter Wahrung der festgesetzten Rangordnung ihrer Elemente gegenseitig eindeutig aufeinander abbilden, oder wie man sich gewöhnlich ausdrückt, aufeinander abzählen lassen." [G. Cantor, letter to W. Wundt (5 Oct 1883)]

Gruß, WM

Carlos Naplos

unread,
May 19, 2022, 10:12:39 AM5/19/22
to


Am 17.05.2022 um 14:48 schrieb Ganzhinterseher:
> Die Mengenlehre erfordert also dass man nicht genau so viele natürliche Zahlen wie Ganzzahlbrüche annehmen darf?

Nein. Jede Lehre erfordert, dass man ein Wort, z.B. "genau so viele
wie", nicht verwendet, wenn seine Bedeutung nicht für alle Beteiligten
in Abhängigkeit vom Geltungsbereich eindeutig ist.

CN

Ganzhinterseher

unread,
May 19, 2022, 10:21:52 AM5/19/22
to
Jede Lehre, die nicht zulässt, dass man genau so viele natürliche Zahlen wie natürliche Zahlen sagt (und demzufolge auch genau so viele natürliche Zahlen wie Ganzzahlbrüche) ist keine Lehre mit mathematischem Gehalt.

Gruß, WM

Ganzhinterseher

unread,
May 19, 2022, 10:25:04 AM5/19/22
to
Juergen Ilse schrieb am Mittwoch, 18. Mai 2022 um 23:20:45 UTC+2:
> Ganzhinterseher <askas...@gmail.com> wrote:
> > Gus Gassmann schrieb am Mittwoch, 18. Mai 2022 um 12:25:23 UTC+2:
> >> Bei jedem endlich indizierten Schritt sind aleph_0 Brüche noch nicht abgearbeitet, und es bleiben aleph_0 Ganzzahlbrüche in der ersten Spalte, um sie zu indizieren.
> >
> > Richtig, nur ist aleph_0 keine quantitative Angabe.
> Selbstverstaendlich ist aleph0 eine "quantitative Angabe" wie jede andere
> Kardinalzahl auch.
>
Das ist falsch, da zum Beispiel: aleph_0 + aleph_0 = aleph_0.

Gruß, WM

Ganzhinterseher

unread,
May 19, 2022, 10:30:01 AM5/19/22
to
Juergen Ilse schrieb am Mittwoch, 18. Mai 2022 um 23:18:24 UTC+2:
> Hallo,
> Gus Gassmann <horand....@gmail.com> wrote:
> > On Tuesday, 17 May 2022 at 17:23:53 UTC-3, Ganzhinterseher wrote:

> Ein "unendlicher schrittweiser Prozess" ist *niemals* beendet. Das ist der
> Grund, weshalb man Abbildungen unendlicher Mengen eben *nicht* als Prozess
> sieht, sondern als eine Teilmenge des kartesischen Produkts von Definitions-
> menge und Bildmenge.

Also ist die Teilmenge, obwohl unendlich, doch beendet?
>
> WM ist offenbar nicht in der Lage, zu begreifen, dass Abbildungen unend-
> licher Menge *niemals* als "Prozess" angesehen werden koennen

Cantor auch nicht: "und es erfährt daher der aus unsrer Regel resultierende Zuordnungsprozeß keinen Stillstand." [Cantor, p. 239]

> und eben etwas
> *voellig* *anderes* sind.

Kann man bei einer Bijektion mit |N irgendeine natürliche Zahl nicht aufsuchen? Kann man irgendeinen Vorgänger nicht finden? Bis zu welcher definierbaren Zahl kann man denn nicht zählen?

Gruß, WM

Gus Gassmann

unread,
May 19, 2022, 10:32:19 AM5/19/22
to
On Thursday, 19 May 2022 at 11:25:04 UTC-3, Ganzhinterseher wrote:
> Juergen Ilse schrieb am Mittwoch, 18. Mai 2022 um 23:20:45 UTC+2:
[...]
> > Selbstverstaendlich ist aleph0 eine "quantitative Angabe" wie jede andere
> > Kardinalzahl auch.
> >
> Das ist falsch, da zum Beispiel: aleph_0 + aleph_0 = aleph_0.

Womit mal wieder bewiesen ist, dass man ganz hinterm See nicht einmal weiss, was eine "quantitative Angabe" überhaupt ist.

Carlos Naplos

unread,
May 19, 2022, 10:38:08 AM5/19/22
to
Auch wenn eine Lehre "zulässt, dass man genau so viele natürliche Zahlen
wie natürliche Zahlen sagt", hat sie keinen Gehalt, wenn nicht klar ist,
welche Bedeutung etwa "genau so viele wie" hat.

Ganzhinterseher

unread,
May 19, 2022, 11:22:23 AM5/19/22
to
Es gibt zu dieser Frage keine Antwort, weil unendliche Mengen nur potentiell unendlich viele definierbare Elemente besitzen. (Zu definierbar siehe https://www.hs-augsburg.de/~mueckenh/Transfinity/Transfinity/pdf) Um aber mein Argument anzuwenden, gehe ich von einer Bijektion aller natürlichen Zahlen mit allen Ganzzahlbrüchen im Sinne der Mengenlehre aus. Dann zeige ich, dass die Menge der positiven Brüche nicht in Bijektion mit diesen natürlichen Zahlen stehen kann, weil für jeden indizierten ein anderer seinen Index verliert. (Das häufigste Gegenargument ist, dass das nur für endlich viele Schritte gilt, im Unendlichen sich aber alles richten wird. Das akzeptiere ich nicht.) Da aber alle definierbaren Brüche einen Index erhalten, müssen die Brüche ohne Index undefinierbar sein. Also gibt es dunkle Zahlen. Damit fällt natürlich auch die Annahme der Gleichzahligkeit von natürlichen Zahlen und Ganzzahlbrüchen. Aber das stört dann ja nicht mehr.

Gruß, WM


Message has been deleted

Gus Gassmann

unread,
May 19, 2022, 2:37:26 PM5/19/22
to
On Thursday, 19 May 2022 at 11:38:08 UTC-3, Carlos Naplos wrote:
[...]
> Auch wenn eine Lehre "zulässt, dass man genau so viele natürliche Zahlen
> wie natürliche Zahlen sagt", hat sie keinen Gehalt, wenn nicht klar ist,
> welche Bedeutung etwa "genau so viele wie" hat.

Hmmmh. Eine Leere also... Der Oberleerer von ganz hinterm See, eben.

Fritz Feldhase

unread,
May 19, 2022, 2:42:07 PM5/19/22
to
On Thursday, May 19, 2022 at 4:21:52 PM UTC+2, Ganzhinterseher wrote:

> Jede Lehre, die nicht zulässt, dass man "genau so viele natürliche Zahlen wie natürliche Zahlen" sagt (und demzufolge auch "genau so viele natürliche Zahlen wie Ganzzahlbrüche") ist keine Lehre mit mathematischem Gehalt.

Das mag schon sein, Mückenheim. Nur muss man halt im Kontext der Mathematik DEFINIEREN, was man mit "genau so viele ... wie ..." meint.

Frege hat z. B. in seinem Buch "Grundlagen der Arithmetik" (1884) den Begriff "gleichzahlig" definiert. Mithilfe dieses Begriffs könnte man z. B. das normalsprachliche "Es gibt genau so viele natürliche Zahlen wie natürliche Zahlen" (im Kontext der Mengenlehre) wie folgt mathematisch präzise ausdrücken: "Die Menge der natürlichen Zahlen und die Menge der natürlichen Zahlen sind gleichzahlig."

Eine andere Vorgehensweise wäre mit Cantor die Mächtigkeiten (Kardinalzahlen) von Mengen zu betrachten und zu DEFINIEREN:

A hat genau so viele Elemente wie B :<-> card(A) = card(B) (wo A und B Mengen sind).

Es würde dann daraus (wegen card(IN) = card(IN)) sofort folgen: IN hat genau so viele Elemente wie IN.

Jedoch darf man sich dann NICHT daran stören, dass DANN - entsprechend der von uns gewählten DEFINITION - (wegen card(A) = card(B) <-> A ~ B) auch gilt:

{1, 2, 3, 4, ...} hat genau so viele Elemente wie {2, 4, 6, ...}.

Oder: {0, 1, 2, 3, 4, ...} hat genau so viele Elemente wie {1, 2, 3, 4, ...}.

Ich persönlich sehe darin aber kein Problem. Wir sagen/behaupten ja auch nicht, dass {1, 2, 3, 4, ...} genau DIESELBEN Elemente enthält, wie {2, 4, 6, ...}, oder dass 0, 1, 2, 3, 4, ...} genau DIESELBEN Elemente enthält wie {1, 2, 3, 4, ...}.

Jedenfalls haben wir dann "genau so viele natürliche Zahlen wie Ganzzahlbrüche" und "genau so viele natürliche Zahlen wie Brüche". (Dass Jürgen I. den Ausdruck "genau so viele" nicht verwenden will, wenn er sich auf unendliche "Gesamtheiten" bezieht, sei ihm unbenommen. Dennoch kann man in diesem Zusammenhang auch den oben beschriebenen Sprachgebrauch einführen. Du bist natürlich für beides zu blöde: Weder verstehst Du, warum Jürgen diesen Sprachgebrauch ablehnt, noch warum man die entsprechenden Ausdrücke DEFINIEREN muss, wenn man sie in diesem Zusammenhang verwenden möchte.)

Fritz Feldhase

unread,
May 19, 2022, 2:45:20 PM5/19/22
to
On Thursday, May 19, 2022 at 4:25:04 PM UTC+2, Ganzhinterseher wrote:
> Juergen Ilse schrieb am Mittwoch, 18. Mai 2022 um 23:20:45 UTC+2:
> > Ganzhinterseher <askas...@gmail.com> wrote:
> > >
> > > [...] nur ist aleph_0 keine quantitative Angabe.
> > >
> > Selbstverstaendlich ist aleph0 eine "quantitative Angabe" wie jede andere Kardinalzahl auch.

In der Tat.

> Das ist falsch, da zum Beispiel: aleph_0 + aleph_0 = aleph_0.

Ach, halt doch mal die Klappe, Du Depp.

Tom Bola

unread,
May 19, 2022, 4:30:42 PM5/19/22
to
Fritz Feldhase schrieb:
Eben - beide raus aus eurem Hamsterrad!

Juergen Ilse

unread,
May 19, 2022, 4:59:31 PM5/19/22
to
Hallo,

Ganzhinterseher <askas...@gmail.com> wrote:
> Es gibt zu dieser Frage keine Antwort, weil unendliche Mengen nur potentiell unendlich viele definierbare Elemente besitzen.

... und das ist schliesslich unzweifelhaft, wil IHRE mathemagische Heiligkeit
Herr "VonGanzHintenGarNixVersteher", geistiger Vater seiner Matheologie,
es so gesagt hat.

Tschuess,
Juergen Ilse (jue...@usenet-verwaltung.de)

Juergen Ilse

unread,
May 19, 2022, 5:07:13 PM5/19/22
to
Hallo,

Fritz Feldhase <franz.fri...@gmail.com> wrote:
> Jedenfalls haben wir dann "genau so viele natürliche Zahlen wie Ganzzahlbrüche" und "genau so viele natürliche Zahlen wie Brüche". (Dass Jürgen I. den Ausdruck "genau so viele" nicht verwenden will, wenn er sich auf unendliche "Gesamtheiten" bezieht, sei ihm unbenommen. Dennoch kann man in diesem Zusammenhang auch den oben beschriebenen Sprachgebrauch einführen. Du bist natürlich für beides zu blöde: Weder verstehst Du, warum Jürgen diesen Sprachgebrauch ablehnt, noch warum man die entsprechenden Ausdrücke DEFINIEREN muss, wenn man sie in diesem Zusammenhang verwenden möchte.)

Der Grund, weshalb ich den Begriff nicht verwenden will, ist, dass er mit
einer (nicht fuer unendliche Mengen definierten) "umgangssprachlichen
Bedeutung" (die sich nur auf endliche Mengen bezieht) "vorbelastet ist,
und deswegen schlichte Gemueter wie WM sich deswegen dazu bewogen fuehlen,
auf eine (neue) Definition des Begriffs, die *auch* unendliche Mengen um-
fasst, zu verziehcten. Der Begriff verwirrt mehr, als dass er nuetzt, ins-
besondere, wenn auf die (notwendige) Definition verzichtet wird.

Die (umgangsspracchliche) Bedeutung des Begriffs "genau so viele" ist im
Zusammenhang mit Unendlichkeit in der Tat hirnrissig und nicht anwendbar.

Tschuess,
Juergen Ilse (jue...@usenet-verwaltung.de)

Juergen Ilse

unread,
May 19, 2022, 5:15:34 PM5/19/22
to
Hallo,
Na und? 0+0=0. Ist deswegen 0 keine Kardinalzahl? Wenn nein, wasist dann
die Kardinalitaet der leeren Menge?

Tschuess,
Juergen Ilse (jue...@usenet-verwaltung.de)

Juergen Ilse

unread,
May 19, 2022, 5:20:34 PM5/19/22
to
Hallo,

Ganzhinterseher <askas...@gmail.com> wrote:
> Juergen Ilse schrieb am Mittwoch, 18. Mai 2022 um 23:18:24 UTC+2:
>> Hallo,
>> Gus Gassmann <horand....@gmail.com> wrote:
>> > On Tuesday, 17 May 2022 at 17:23:53 UTC-3, Ganzhinterseher wrote:
>
>> Ein "unendlicher schrittweiser Prozess" ist *niemals* beendet. Das ist der
>> Grund, weshalb man Abbildungen unendlicher Mengen eben *nicht* als Prozess
>> sieht, sondern als eine Teilmenge des kartesischen Produkts von Definitions-
>> menge und Bildmenge.
>
> Also ist die Teilmenge, obwohl unendlich, doch beendet?

Welche Teilmenge? Und was soll eine "beendete Menge" sein? Was eine "nicht
beendete Menge"? Und Beitte kein Geschwurbel sondern Definitionen.

Tschuess,
Juergen Ilse (jue...@usenet-verwaltung.de)

Juergen Ilse

unread,
May 19, 2022, 5:25:00 PM5/19/22
to
Hallo,

Ganzhinterseher <askas...@gmail.com> wrote:
> Fritz Feldhase schrieb am Mittwoch, 18. Mai 2022 um 21:16:13 UTC+2:
>
>> Hinweis: Für jedes Glied a in der Folge der natürlichen Zahlen gibt es ein Glied b (in der Folge), das größer ist als a. Es gibt aber kein Glied b (in der Folge), das größer ist als alle Glieder a in der Folge. Daraus kann man nun NICHT schließen, dass es natürliche Zahlen gibt, die nicht Glieder der Folge aller natürlicher Zahlen (in natürlicher Ordnung) sind.
>
> Die Quantorenvertauschung ist hier offensichtlich falsch. Sie ist aber nicht immer falsch.

Quantorenvertauschung (bzw. Aussagen, die aus einer solchen hervorgegangen
sind) haben aber *NIEMALS* irgend eine Beweiskraft.

> Im folgenden Beispiel ist sie offensichtlich richtig:
>
> Es gibt ℵo Nachfolger zwischen jeder definierbaren natürlichen Zahl n und ω.
> Also existiert keine def. natürliche Zahl mit weniger Nachfolgern zwischen ihr und ω.
> Das impliziert,

... ueberhaupt nichts, weil eine durch Quantorenvertauschung entstandene
Behauptung ohne epiliziten *anderen* Beweis keinerlei Beweiskraft hat.

Tschuess,
Juergen Ilse (jue...@usenet-verwaltung.de)

Juergen Ilse

unread,
May 19, 2022, 5:31:18 PM5/19/22
to
Hallo,

Ganzhinterseher <askas...@gmail.com> wrote:
> Fritz Feldhase schrieb am Mittwoch, 18. Mai 2022 um 17:01:03 UTC+2:
>> On Wednesday, May 18, 2022 at 4:05:36 PM UTC+2, Ganzhinterseher wrote:
>
>> > Sie enthält alle Brüche, auf die noch unendlich viele folgen, doch diese unendlich vielen enthält sie nicht, denn dann würden sie ja nicht mehr folgen.
>> Das ist ein bestechendes Argument! Demnach entält auch die Folge aller natürlichen Zahlen (1, 2, 3, ...) nicht alle natürlichen Zahlen?
>
> Wenn es nach Dir ginge, dann nicht.
> Es gibt ℵo Nachfolger zwischen jeder natürlichen Zahl n und ω.
> Also existiert keine natürliche Zahl mit weniger Nachfolgern zwischen ihr und ω.

So weit korrwekt.

> Das impliziert, dass zwischen allen Elementen der Menge der natürlichen Zahlen und ω ℵo Nachfolger existieren.

Nein. Zwischem *JEDEM* (bliebig aber festen) Element der natuerlihen Zahlen
und omega (wenn man denn diese Formulierung akzeptiert: ich wuerde eher sagen
"mit unendlich vielen natuerlichen Nachfolgern).

> Das impliziert, dass zwischen der Menge der natürlichen Zahlen und ω ℵo Nachfolger existieren.

Nein, das folgt *nicht*. Diese Schlussfolgerung existiert ausschliesslich in
IHREM geistigen Wahnsystem, dass SIE fuer Mathematik halten. Deswegen koennen
SIE diese Schlussfolgerung auch niicht mit mathematischen Mitteln beweisen.

Tschuess,
Juergen Ilse (jue...@usenet-verwaltung.de)

Fritz Feldhase

unread,
May 19, 2022, 9:31:14 PM5/19/22
to
On Thursday, May 19, 2022 at 11:07:13 PM UTC+2, Juergen Ilse wrote:
> Fritz Feldhase wrote:
> >
> > Jedenfalls haben wir dann "genau so viele natürliche Zahlen wie Ganzzahlbrüche" und "genau so viele natürliche Zahlen wie Brüche". (Dass Jürgen I. den Ausdruck "genau so viele" nicht verwenden will, wenn er sich auf unendliche "Gesamtheiten" bezieht, sei ihm unbenommen. Dennoch kann man in diesem Zusammenhang auch den oben beschriebenen Sprachgebrauch einführen. Du bist natürlich für beides zu blöde: Weder verstehst Du, warum Jürgen diesen Sprachgebrauch ablehnt, noch warum man die entsprechenden Ausdrücke DEFINIEREN muss, wenn man sie in diesem Zusammenhang verwenden möchte.)
> >
> Der Grund [...]

Das brachst Du mir nicht zu erklären. Allerdings ist die Begründung einigermaßen haltlos. Es gibt eine ganze Reihe von Begriffen, die im Kontext der Mathematik eine andere Bedeutung besitzen als in der "Umgangssprache". Eben darum definiert man diese ja auch im Kontext der Mathematik _explizit_. Das kann man z. B. auch mit dem Begriff "Anzahl" machen.

Im angelsächsischen Bereich scheint man da keine solchen Berührungsängste zu haben, wie Du (und auch andere) sie an den Tag legst (legen):

"In mathematics, the cardinality of a set is a measure of the "number of elements" of the set." (Wikipedia, Cardinality)

Oder: "Cardinal arithmetic can be used to show not only that the number of points in a real number line is equal to the number of points in any segment of that line, but that this is equal to the number of points on a plane and, indeed, in any finite-dimensional space." (Wikipedia, Cardinality)

Im deutschen Sprachraum scheint man da etwas mehr Gewicht darauf zu legen, den Begriff "Anzahl" in diesem Zusammenhang zu vermeiden und von "Mächtigkeit" zu sprechen:

"In der Mathematik verwendet man den aus der Mengenlehre von Georg Cantor stammenden Begriff der Mächtigkeit oder Kardinalität, um den für endliche Mengen verwendeten Begriff der „Anzahl der Elemente einer Menge“ auf unendliche Mengen zu verallgemeinern." (Wikipedia, Mächtigkeit)

Tatsächlich hat Cantor den Begriff der "Anzahl" idiotischerweise in Zusammenhang mit dem ORDINALZAHLBEGRIFF verwendet. D a s hat sich selbstverständlich NICHT durchgesetzt, so dass der Begriff hierzulande leider häufig gleich ganz vermieden wird, wenn es um unendliche Mengen geht. (Wie man an obigem Zitat "Cardinal arithmetic can be used to ..." sieht, geht es offenbar auch anders.)
Message has been deleted

Fritz Feldhase

unread,
May 19, 2022, 10:14:44 PM5/19/22
to
On Tuesday, May 17, 2022 at 4:10:10 PM UTC+2, Fritz Feldhase wrote:
> On Tuesday, May 17, 2022 at 3:55:36 PM UTC+2, Fritz Feldhase wrote:
> > On Monday, May 16, 2022 at 7:13:40 PM UTC+2, Ganzhinterseher wrote
> >
> > WM betrachtet die Matrix M1:
> >
> > 1/1, 1/2, 1/3, ...
> > 2/1, 2/2, 2/3, ...
> > 3/1, 3/2, 3/3, ...
> > 4/1, 4/2, 4/3, ...
> > ...
> >
> > Diese ist wie folgt definiert:
> >
> > M1_n,m = n/m für alle n,m e IN.

Man kann zudem auch die folgende Matrix M2 betrachten:

1, _3, _5, _7, ...
2, _6, 10, 14, ...
4, 12, 20, 28, ...
8, 24, 40, 56, ...
...

Diese ist wie folgt definiert:

M2_n,m = 2^(n-1)*(2m-1) für alle n,m e IN.

Dass 2^(n-1)*(2m-1) für alle n, m e IN eine natürliche Zahl ist, ist klar.

Wie aber sieht es mit der "Umkehrung" aus? Sei k eine beliebige natürliche Zahl. Gibt es dann zu k zwei Zahlen n, m e IN, so dass k = 2^(n-1)*(2m-1) ist?

Fall 1: k ist ungerade. Dann gibt es per definitionem ein m e IN, so dass k = 2m-1 ist. (Wir setzen hier mit Mückenheim IN = {1, 2, 3, ...} voraus.) Es gibt dann mit n = 1 zwei Zahlen n, m e IN, so dass k = 2^(n-1)*(2m-1) ist.

Fall 2: k ist gerade. Dann besitzt k einen Teiler der Form 2^p mit p e IN. Sei q die größte natürliche Zahl, so dass 2^q ein Teiler von k ist. Wir setzen r = k / 2^q und m = (r + 1) / 2. Dann sind n = q+1 und m zwei natürliche Zahlen, so dass k = 2^(n-1)*(2m-1) gilt.

Dass n, m zu gegebenem k eindeutig bestimmt sind, kann leicht gezeigt werden. :-P

Quelle: https://de.wikipedia.org/wiki/Cantorsche_Paarungsfunktion#Alternativen

Juergen Ilse

unread,
May 20, 2022, 1:02:36 AM5/20/22
to
Fritz Feldhase <franz.fri...@gmail.com> wrote:
> "In der Mathematik verwendet man den aus der Mengenlehre von Georg Cantor stammenden Begriff der Mächtigkeit oder Kardinalität, um den für endliche Mengen verwendeten Begriff der „Anzahl der Elemente einer Menge“ auf unendliche Mengen zu verallgemeinern." (Wikipedia, Mächtigkeit)
>
> Tatsächlich hat Cantor den Begriff der "Anzahl" idiotischerweise in Zusammenhang mit dem ORDINALZAHLBEGRIFF verwendet. D a s hat sich selbstverständlich NICHT durchgesetzt, so dass der Begriff hierzulande leider häufig gleich ganz vermieden wird, wenn es um unendliche Mengen geht. (Wie man an obigem Zitat "Cardinal arithmetic can be used to ..." sieht, geht es offenbar auch anders.)

... und weil sich der Begriff Anzahl in Umgangssprache und bei Caantor bereits
erheblich unterscheiden, ist es IMHO unklug, dem Begriff (zur Verwirrung aller
Leser) noch eine dritte, von beiden Beeutungen *verschiedene*, neue Bedeutung
hinzuzufuegen.

Es ist ja schon so schlimm genug, dass WM dauernd Cantor mit voellig
unpassenden Zitaten heranzieht (ueber "Abbildungen von Wohlordnungen
unter Wahrung der Ordnung", wenn von Kardinalitaeten die Rede ist).

Tschuess,
Juergen Ilse (jue...@usenet-verwaltung.de)

Ganzhinterseher

unread,
May 20, 2022, 2:06:05 PM5/20/22
to
Juergen Ilse schrieb am Freitag, 20. Mai 2022 um 07:02:36 UTC+2:

> Es ist ja schon so schlimm genug, dass WM dauernd Cantor mit voellig
> unpassenden Zitaten heranzieht (ueber "Abbildungen von Wohlordnungen
> unter Wahrung der Ordnung", wenn von Kardinalitaeten die Rede ist).

Mit Bezug auf die natürlichen Zahlen ist das die Definition der Kardinalität. "Zwei wohlgeordnete Mengen M und N heissen von gleichem Typus oder auch von gleicher Anzahl, wenn sie sich gegenseitig eindeutig und vollständig unter beidseitiger Wahrung der Rangfolge ihrer Elemente auf einander beziehen, abbilden lassen;" [G. Cantor, letter to R. Lipschitz (19 Nov 1883)] Einige dieser Anzahlen gehören zu einer Zahlenklasse, der die Mächtigkeit aleph_0 zugeschrieben wird.

Gruß, WM

Ganzhinterseher

unread,
May 20, 2022, 2:11:29 PM5/20/22
to
Fritz Feldhase schrieb am Donnerstag, 19. Mai 2022 um 20:42:07 UTC+2:

> Jedoch darf man sich dann NICHT daran stören, dass DANN - entsprechend der von uns gewählten DEFINITION - (wegen card(A) = card(B) <-> A ~ B) auch gilt:
>
> {1, 2, 3, 4, ...} hat genau so viele Elemente wie {2, 4, 6, ...}.
>
> Oder: {0, 1, 2, 3, 4, ...} hat genau so viele Elemente wie {1, 2, 3, 4, ...}.

Wie ich in meiner Antwort an CN schon erwähnte, verwende ich als Ausgangspunkt Cantors Bijektion, wonach Ganzzahlbrüche und natürliche Zahlen genau gleich viele Elemente haben. Man könnte allerdings auch feststellen, dass n/1 und n wertmäßig identisch sind, womit sich die Gleichzahligkeit sofort ergibt.

Gruß, WM

Ganzhinterseher

unread,
May 20, 2022, 2:15:57 PM5/20/22
to
Juergen Ilse schrieb am Donnerstag, 19. Mai 2022 um 23:20:34 UTC+2:
> Hallo,
> Ganzhinterseher <askas...@gmail.com> wrote:
> > Juergen Ilse schrieb am Mittwoch, 18. Mai 2022 um 23:18:24 UTC+2:
> >> Hallo,
> >> Gus Gassmann <horand....@gmail.com> wrote:
> >> > On Tuesday, 17 May 2022 at 17:23:53 UTC-3, Ganzhinterseher wrote:
> >
> >> Ein "unendlicher schrittweiser Prozess" ist *niemals* beendet. Das ist der
> >> Grund, weshalb man Abbildungen unendlicher Mengen eben *nicht* als Prozess
> >> sieht, sondern als eine Teilmenge des kartesischen Produkts von Definitions-
> >> menge und Bildmenge.
> >
> > Also ist die Teilmenge, obwohl unendlich, doch beendet?
> Welche Teilmenge?

Die oben erwähnte.

> Und was soll eine "beendete Menge" sein?

Das ist, was Cantor fertig nennt:

"Die Totalität aller Alefs ist nämlich eine solche, welche nicht als eine bestimmte, wohldefinirte fertige Menge aufgefaßt werden kann. [...] 'Wenn eine bestimmte wohldefinirte fertige Menge eine Cardinalzahl haben würde, die mit keinem der Alefs zusammenfiele, so müßte sie Theilmengen enthalten, deren Cardinalzahl irgend ein Alef ist, oder mit anderen Worten, die Menge müßte die Totalität aller Alefs in sich tragen.' Daraus ist leicht zu folgern, daß unter der eben genannten Voraussetzung (einer best. Menge, deren Cardinalzahl kein Alef wäre) auch die Totalität aller Alefs als eine best. wohldefinirte fertige Menge aufgefaßt werden könnte." [G. Cantor, letter to D. Hilbert (26 Sep 1897)]

"In meinen Untersuchungen habe ich, allgemein gesprochen, 'fertige Mengen' im Auge und verstehe darunter solche, bei denen die Zusammenfassung aller Elemente zu einem Ganzen, zu einem Ding für sich möglich ist, so daß eine 'fertige M.' eventuell selbst als Element einer andern Menge gedacht werden kann. [...] Derartige Mengen, die die Bedingung 'fertig' nicht erfüllen, nenne ich 'absolut unendliche' Mengen.
Nehmen wir einmal an, es könnten alle Alefs coexistieren, so führt uns dies zu einem Widerspruch. Denn alsdann würden alle Alefs, wenn wir sie nach ihrer Größe geordnet denken, eine wohlgeordnete, fertige Menge M bilden. Mit jeder wohlgeordneten fertigen Menge M von Alefs ist aber nach dem Bildungsgesetz der Alefs ein bestimmtes Alef gegeben, welches der Größe nach auf alle Individuen von M nächstfolgt.
Hier hätten wir also den Widerspruch eines Alefs, das größer wäre als alle Alefs, folglich auch größer als es selbst. Ich schließe also, daß alle Alefs nicht coexistent sind, nicht zu einem 'Ding für sich' zusammengefasst werden können, daß sie mit anderen Worten keine 'fertige Menge' bilden.
Der Widerspruch erscheint mir so, als wenn wir von einer 'endlichen Zahl' sprechen wollten, die größer wäre als 'alle endlichen Zahlen'. Nur ist hier der Unterschied, daß alle endlichen Zahlen eine fertige Menge bilden, die nach oben von der kleinsten transfiniten Cardinalzahl 0 gewissermaßen begrenzt wird. Die absolute Grenzenlosigkeit der Menge aller Alefs erscheint als Grund der Unmöglichkeit, sie zu einem Ding für sich zusammenzufassen.
In dem von Ihnen vorgetragenen Beispiele wird aber die Menge aller Alefs als eine 'fertige M.' vorausgesetzt und damit löst und erklärt sich der Widerspruch, auf den Sie durch Anwendung von Sätzen geführt werden, die nur für fertige Mengen bewiesen und gültig sind." [G. Cantor, letter to D. Hilbert (6 Oct 1898)]

"Aus der Definition: 'Unter einer fertigen Menge verstehe man jede Vielheit, bei welcher alle Elemente ohne Widerspruch als zusammenseiend und daher als ein Ding für sich gedacht werden können.' ergeben sich mancherlei Sätze, unter Anderm diese:
I 'Ist M eine fert. Menge, so ist auch jede Theilmenge von M eine fert. Menge.'
II 'Substituirt man in einer fert. M. an Stelle der Elemente fertige Mengen, so ist die hieraus resultirende Vielheit eine fertige M.'
III 'Ist von zwei aequivalenten Vielheiten die eine eine fert. M., so ist es auch die andere.'
IV 'Die Vielheit aller Theilmengen einer fertigen Menge M ist eine fertige Menge.' Denn alle Theilmengen von M sind 'zusammen' in M enthalten; der Umstand, daß sie sich theilweise decken, schadet hieran nichts.
Daß die 'abzählbaren' Vielheiten {} fertige Mengen sind, scheint mir ein axiomatisch sicherer Satz zu sein, auf welchem die ganze Functionentheorie beruht. Dagegen scheint mir der Satz 'Das Linearcontinuum ist eine fertige Menge' ein beweisbarer Satz zu sein und zwar so: Das Linearcont. ist aequivalent der Menge S = {f()} wo f() die Werthe 0 oder 1 haben kann. [...] Ich behaupte also S ist eine 'fertige Menge'. [...] Nach Satz IV ist aber die Vielheit aller Theilmengen von {} eine fertige Menge; dasselbe gilt also nach Satz III auch für S und für das Linearcontinuum.
Ebenso dürfte das Prädicat 'fertig' für die Mengen 1, 2, ... beweisbar sein." [G. Cantor, letter to D. Hilbert (10 Oct 1898)]

"Unter Bezugnahme auf mein Schreiben v. 10ten, stellt sich bei genauerer Erwägung heraus, daß der Beweis des Satzes IV keineswegs so leicht geht. Der Umstand, daß die Elemente der 'Vielheit aller Theilmengen einer fertigen Menge' sich theilweise decken, macht ihn illusorisch. In die Definition der fert. Menge wird die Voraussetzung des Getrenntseins resp. Unabhängigseins der Elemente als wesentlich aufzunehmen sein." [G. Cantor, letter to D. Hilbert (12 Oct 1898)]

"Ich habe mich jetzt daran gewöhnt, das was ich früher 'fertig' genannt, durch den Ausdruck 'consistent' zu ersetzen;" [G. Cantor, letter to D. Hilbert (9 May 1899)]

"Die Totalität der Alefs lässt sich nicht als eine bestimmte fertige Menge auffassen." [G. Cantor, letter to A. Schönflies via D. Hilbert (28 Jun 1899)]

"Eine Vielheit kann nämlich so beschaffen sein, daß die Annahme eines 'Zusammenseins' aller ihrer Elemente auf einen Widerspruch führt, so daß es unmöglich ist, die Vielheit als eine Einheit, als 'ein fertiges Ding' aufzufassen. Solche Vielheiten nenne ich absolut unendliche oder inkonsistente Vielheiten." [G. Cantor, letter to R. Dedekind (3 Aug 1899)]

"Zu Elementen einer Vielheit, können nur fertige Dinge genommen werden, nur Mengen, nicht aber inconsistente Vielheiten, in deren Wesen es liegt, daß sie nie als fertig und actuell existirend gedacht werden kann." [G. Cantor, letter to P. Jourdain (9 Jul 1904)]

"'Unter einer Menge verstehen wir jede Zusammenfassung von ... zu einem Ganzen', worin doch liegt, daß Vielheiten, denen das Gepräge des fertigen Ganzen oder der Dinglichkeit nicht nachgesagt werden kann, nicht als 'Mengen' im eigentlichen Sinne des Wortes anzusehen sind." [G. Cantor, letter to G. Chisholm-Young (9 Mar 1907)]

Gruß, WM

Ganzhinterseher

unread,
May 20, 2022, 2:25:57 PM5/20/22
to
Juergen Ilse schrieb am Donnerstag, 19. Mai 2022 um 23:25:00 UTC+2:
> Hallo,
> Ganzhinterseher <askas...@gmail.com> wrote:
> > Fritz Feldhase schrieb am Mittwoch, 18. Mai 2022 um 21:16:13 UTC+2:
> >
> >> Hinweis: Für jedes Glied a in der Folge der natürlichen Zahlen gibt es ein Glied b (in der Folge), das größer ist als a. Es gibt aber kein Glied b (in der Folge), das größer ist als alle Glieder a in der Folge. Daraus kann man nun NICHT schließen, dass es natürliche Zahlen gibt, die nicht Glieder der Folge aller natürlicher Zahlen (in natürlicher Ordnung) sind.
> >
> > Die Quantorenvertauschung ist hier offensichtlich falsch. Sie ist aber nicht immer falsch.
> Quantorenvertauschung (bzw. Aussagen, die aus einer solchen hervorgegangen
> sind) haben aber *NIEMALS* irgend eine Beweiskraft.

Falsch. Jede natürliche Zahl ist kleiner als omega. omega ist größer als jede natürliche Zahl.
∀n ∈ ℕ_def ∃ω: ω > n ∃ω ∀n ∈ ℕ_def: ω > n

Dasselbe gilt für den Bereich unmittelbar links von ω auf der Ordninalzahlachse, von dem Du nichts weißt.

Gruß, WM

Ganzhinterseher

unread,
May 20, 2022, 2:40:12 PM5/20/22
to
Fritz Feldhase schrieb am Freitag, 20. Mai 2022 um 04:14:44 UTC+2:

> Dass n, m zu gegebenem k eindeutig bestimmt sind, kann leicht gezeigt werden. :-P
>
Dass nicht alle Brüche indiziert werden, kann ebenfalls leicht gezeigt werden. In der Matrix

1/1, 1/2, 1/3, 1/4, ...
2/1, 2/2, 2/3, 2/4, ...
3/1, 3/2, 3/3, 3/4, ...
4/1, 4/2, 4/3, 4/4, ...
5/1, 5/2, 5/3, 5/4, ...
...

werden alle Brüche der ersten Spalte indiziert. Nun werden mit diesen Indizes alle Brüche der Cantor-Folge 1/1, 1/2, 2/1, 1/3, 2/2, 3/1, 1/4, 2/3, 3/2, 4/1, 1/5, ... indiziert. Das bedeutet, dass fast alle Brüche nicht indiziert werden. Verstanden?

Gruß, WM

Juergen Ilse

unread,
May 20, 2022, 2:42:40 PM5/20/22
to
Hallo,

Ganzhinterseher <askas...@gmail.com> wrote:
> Juergen Ilse schrieb am Freitag, 20. Mai 2022 um 07:02:36 UTC+2:
>
>> Es ist ja schon so schlimm genug, dass WM dauernd Cantor mit voellig
>> unpassenden Zitaten heranzieht (ueber "Abbildungen von Wohlordnungen
>> unter Wahrung der Ordnung", wenn von Kardinalitaeten die Rede ist).
>
> Mit Bezug auf die natürlichen Zahlen ist das die Definition der Kardinalität.

Unfug. "Gleichmaechtigkeit" setzt *NIEMALS* irgend eine Ordnung auf den
beteiligten Mengen voraus, erst recht keine Wohlornung. Die zitierte Text-
stelle von Cantor bezieht sich nicht auf Aequivalenz von Mengen sondern
auf Aehnlichkeit von Mengen (nach Cantorschen Begriffen). Fuer die Aehn-
lichkeit ist in der Tat eine Wohlordnung erforderlich, fuer Aequivalenz
(spich "Gleichmaechtigkeit") jedoch *nicht*.

Tschuess,
Juergen Ilse (jue...@usenet-verwaltung.de)

Juergen Ilse

unread,
May 20, 2022, 2:51:22 PM5/20/22
to
Hallo,
Fuer den Nachweis der "nicht Gleichmaechtigkeit" reicht es nicht aus,
eine nicht bijektive Abbildung zu finden. Man muss dazu nachweisen,
dass es *gar* *keine* nicht bijektive Abbildung zwischen den Mengen
gibt, und das kann man nicht anhand einer (bei unendlichen Mengen
immer existierenden) nicht bijektiven Abbildung zeigen.

Also zeigen SIE, dass die Cantorsche Abbildung *nicht* bijektiv ist.
Die von *IHNEN* genannte Abbildung sagt ueber die Cantorsche Abbildung
*rein* *gar* *nichts* aus.

Tschuess,
Juergen Ilse (jue...@usenet-verwaltung.de)

Juergen Ilse

unread,
May 20, 2022, 2:56:20 PM5/20/22
to
Hallo,

Ganzhinterseher <askas...@gmail.com> wrote:
> Falsch. Jede natürliche Zahl ist kleiner als omega. omega ist größer als jede natürliche Zahl.
> ∀n ∈ ℕ_def ∃ω: ω > n ∃ω ∀n ∈ ℕ_def: ω > n

Wenn nur *ein* Quantor vorkommt, bekommen noch nicht einmal SIE eine
Quantorenvertauschung hin ...

> Dasselbe gilt für den Bereich unmittelbar links von ω auf der Ordninalzahlachse, von dem Du nichts weißt.

omega ist eine "limes Ordinalzahl", sprich es existiert *nichts*
"unmittelbar links von omega". Oder anders formuliert: omega hat
keien direkten Vorgaenger (im Gegensatz zu *JEDER* natuerlichen
Zahl).

Tschuess,
Juergen Ilse (jue...@usenet-verwaltung.de)

Ganzhinterseher

unread,
May 20, 2022, 2:57:07 PM5/20/22
to
Juergen Ilse schrieb am Freitag, 20. Mai 2022 um 20:42:40 UTC+2:
> Hallo,
> Ganzhinterseher <askas...@gmail.com> wrote:
> > Juergen Ilse schrieb am Freitag, 20. Mai 2022 um 07:02:36 UTC+2:
> >
> >> Es ist ja schon so schlimm genug, dass WM dauernd Cantor mit voellig
> >> unpassenden Zitaten heranzieht (ueber "Abbildungen von Wohlordnungen
> >> unter Wahrung der Ordnung", wenn von Kardinalitaeten die Rede ist).
> >
> > Mit Bezug auf die natürlichen Zahlen ist das die Definition der Kardinalität.
> Unfug. "Gleichmaechtigkeit" setzt *NIEMALS* irgend eine Ordnung auf den
> beteiligten Mengen voraus, erst recht keine Wohlornung.

Der Beweis der Gleichmächtigkeit einer Menge M mit |N setzt eine Wohlordnung voraus.

> Die zitierte Text-
> stelle von Cantor bezieht sich nicht auf Aequivalenz von Mengen sondern
> auf Aehnlichkeit von Mengen (nach Cantorschen Begriffen). Fuer die Aehn-
> lichkeit ist in der Tat eine Wohlordnung erforderlich, fuer Aequivalenz
> (spich "Gleichmaechtigkeit") jedoch *nicht*.

Dann zeige die Gleichmächtigkeit von |N und |Q ohne eine Wohlordnung von |Q.

Gruß, WM

Ralf Bader

unread,
May 20, 2022, 3:01:34 PM5/20/22
to
Nein, Mückenheim, das was Sie hier als "beendete Menge"
daherschwadronieren, ist nicht das, was Cantor zeitweilig als "fertige
Menge" bezeichnet hat. Die nicht "fertigen" Mengen sind vielmehr die
"absolut unendlichen" Kollektionen, und das sind, grob gesagt,
diejenigen, bei denen Paradoxien der Art von Burali-Forti auftreten. Das
ist aber noch lange nicht der Fall bei Mengen, die nur ein bißchen
unendlich sind.

Im Übrigen sind Cantorsche Originalschriften inzwischen von primär
historischem Interesse und keine Quellen zur Bearbeitung aktueller
Fragen der Mengenlehre.

Eines der obigen Zitate ist faktisch eine Formulierung des
Ersetzungsaxioms, welches ein Hauptgrund für das F in ZFC wurde. Zermelo
ist das also offenbar bei der, am Cantorschen Theoriebestand
orientierten, Formulierung seiner Axiome entgangen. Ihnen, Mückenheim,
entgeht jedoch nahezu alles. Sie sind für die von Ihnen manisch
beswchwafelten Gegenstände zu blöde.

Tom Bola

unread,
May 20, 2022, 3:02:53 PM5/20/22
to
Juergen Ilse schrieb:

> Ganzhinterseher <askas...@gmail.com> wrote:
>> Falsch. Jede natürliche Zahl ist kleiner als omega. omega ist größer als jede natürliche Zahl.
>> ∀n ∈ ℕ_def ∃ω: ω > n ∃ω ∀n ∈ ℕ_def: ω > n
>
> Wenn nur *ein* Quantor vorkommt, bekommen noch nicht einmal SIE eine
> Quantorenvertauschung hin ...

LOL, gut bebachtet...

Ganzhinterseher

unread,
May 20, 2022, 3:04:50 PM5/20/22
to
Juergen Ilse schrieb am Freitag, 20. Mai 2022 um 20:51:22 UTC+2:

> Fuer den Nachweis der "nicht Gleichmaechtigkeit" reicht es nicht aus,
> eine nicht bijektive Abbildung zu finden. Man muss dazu nachweisen,
> dass es *gar* *keine* nicht bijektive Abbildung zwischen den Mengen
> gibt, und das kann man nicht anhand einer (bei unendlichen Mengen
> immer existierenden) nicht bijektiven Abbildung zeigen.

Ich beweise es, indem ich die Menge der natürlichen Zahlen anhand der ersten Spalte der Matrix festlege. Keine weiter natürliche Zahl darf und kann hinzugezogen werden. Bei den Abbildungen, die Dir behagen, wird diese Beschränkung vergessen.
>
> Also zeigen SIE, dass die Cantorsche Abbildung *nicht* bijektiv ist.

Sofern nur die Cantorsche Folge
1/1, 1/2, 2/1, 1/3, 2/2, 3/1, 1/4, 2/3, 3/2, 4/1, 1/5, ...
betroffen ist, ist die Abbildung bijektiv. Das ist ja, was jeder sieht und glaubt.

Sofern aber alle positiven Brüche betroffen sind, ist keine Bijektion möglich, denn der Anteil der nicht indizierten Brüche bleibt nun einmal unverändert - und mehr Indizes sind nicht verfügbar.

Gruß, WM


Tom Bola

unread,
May 20, 2022, 3:06:28 PM5/20/22
to
Clown WM saicht wieder und wieder entsetzlich verblödeten Stuss:

> zeige die Gleichmächtigkeit von |N und |Q ohne eine Wohlordnung von |Q

Du kennst die Cantors Zuordnung zwischen beiden und die gilt für
jede beliebige Anordnung.

Ralf Bader

unread,
May 20, 2022, 3:06:50 PM5/20/22
to
Mit der offenbaren Verwendung der Konstanten ω am Platz der Variablen in
einem Quantorenausdruck bestätigen Sie erneut, daß Sie für Mathematik zu
blöde sind und nur Scheißdreck daherschwafeln.

Ganzhinterseher

unread,
May 20, 2022, 3:07:24 PM5/20/22
to
Juergen Ilse schrieb am Freitag, 20. Mai 2022 um 20:56:20 UTC+2:
> Ganzhinterseher <askas...@gmail.com> wrote:
> > Falsch. Jede natürliche Zahl ist kleiner als omega. omega ist größer als jede natürliche Zahl.
> > ∀n ∈ ℕ_def ∃ω: ω > n ∃ω ∀n ∈ ℕ_def: ω > n
> Wenn nur *ein* Quantor vorkommt, bekommen noch nicht einmal SIE eine
> Quantorenvertauschung hin ...

Oben stehen zwei ∀ und ∃. Bist Du ganz blind?

> > Dasselbe gilt für den Bereich unmittelbar links von ω auf der Ordninalzahlachse, von dem Du nichts weißt.
> omega ist eine "limes Ordinalzahl", sprich es existiert *nichts*
> "unmittelbar links von omega".

Das ist die Behauptung der Blinden.

Gruß, WM

Tom Bola

unread,
May 20, 2022, 3:09:44 PM5/20/22
to
Clown WM saicht wieder und wieder seinen gleichen Scheissdreck daher:

> und mehr Indizes sind nicht verfügbar

Doch, sind es: per Axiomierung und damit der Fundierung UNSERER Mathematik.

Ganzhinterseher

unread,
May 20, 2022, 3:12:13 PM5/20/22
to
Ralf Bader schrieb am Freitag, 20. Mai 2022 um 21:01:34 UTC+2:

> Nein, Mückenheim, das was Sie hier als "beendete Menge"
> daherschwadronieren, ist nicht das, was Cantor zeitweilig als "fertige
> Menge" bezeichnet hat.

Das ist Dein Irrtum. Denke nochmal nach.

> Die nicht "fertigen" Mengen sind

Es ging bei der Frage um die fertigen Mengen, die ich als beendet tituliert habe, nicht um die unfertigen.
>
> Im Übrigen sind Cantorsche Originalschriften inzwischen von primär
> historischem Interesse und keine Quellen zur Bearbeitung aktueller
> Fragen der Mengenlehre.

Es ist bei allen Religionen so, dass die Gedanken ihrer Begründer später pervertiert werden.

Gruß, WM

Tom Bola

unread,
May 20, 2022, 3:12:33 PM5/20/22
to
Ralf Bader schrieb:

>> ∀n ∈ ℕ_def ∃ω: ω > n ∃ω ∀n ∈ ℕ_def: ω > n

> Mit der offenbaren Verwendung der Konstanten ω am Platz der Variablen in
> einem Quantorenausdruck bestätigen Sie erneut, daß Sie für Mathematik zu
> blöde sind und nur Scheißdreck daherschwafeln.

Bin wirklich mal gespannt, ob dieser Clown diese Feststellung kapiert, ROFL...

Tom Bola

unread,
May 20, 2022, 3:14:49 PM5/20/22
to
Clown WM saicht wie immer abartig unterbelichteten Scheissdreck:

>>> ∀n ∈ ℕ_def ∃ω: ω > n ∃ω ∀n ∈ ℕ_def: ω > n

>> Wenn nur *ein* Quantor vorkommt, bekommen noch nicht einmal SIE eine
>> Quantorenvertauschung hin ...
>
> Oben stehen zwei ∀ und ∃. Bist Du ganz blind?

ROFL, https://en.wikipedia.org/wiki/Quantifier_shift ...

Gus Gassmann

unread,
May 20, 2022, 3:15:44 PM5/20/22
to
On Friday, 20 May 2022 at 15:56:20 UTC-3, Juergen Ilse wrote:
> Hallo,
> Ganzhinterseher <askas...@gmail.com> wrote:
[...]
> Wenn nur *ein* Quantor vorkommt, bekommen noch nicht einmal SIE eine
> Quantorenvertauschung hin ...

Da wäre ich mir nicht so sicher...

Tom Bola

unread,
May 20, 2022, 3:15:57 PM5/20/22
to
Clown WM saicht:
Abartig unterbelichtet...

Tom Bola

unread,
May 20, 2022, 3:20:51 PM5/20/22
to
Gus Gassmann schrieb:
Aber dann bitte ohne Ausdrücke wie " Für alle n in N existiert ein omega... ",
anderenfalls ist es keine _Quantoren_ -Vertauschung sonder WM-Dreck.

Transfinity

unread,
May 20, 2022, 3:20:58 PM5/20/22
to
Ralf Bader schrieb am Freitag, 20. Mai 2022 um 21:06:50 UTC+2:
> On 05/20/2022 08:25 PM, Ganzhinterseher wrote:

> >> Quantorenvertauschung (bzw. Aussagen, die aus einer solchen hervorgegangen
> >> sind) haben aber *NIEMALS* irgend eine Beweiskraft.
> >
> > Falsch. Jede natürliche Zahl ist kleiner als omega. omega ist größer als jede natürliche Zahl.
> > ∀n ∈ ℕ_def ∃ω: ω > n ∃ω ∀n ∈ ℕ_def: ω > n
> >
> > Dasselbe gilt für den Bereich unmittelbar links von ω auf der Ordinalzahlachse, von dem Du nichts weißt.
> Mit der offenbaren Verwendung der Konstanten ω am Platz der Variablen

Der Platz kann ebenso eine Konstante enthalten. Ist der Ausdruck ∀n ∈ ℕ_def ∃ω: ω > n in Deiner Matheologie verboten? In gewöhnlicher Logik nicht.

Gruß, WM

Tom Bola

unread,
May 20, 2022, 3:28:39 PM5/20/22
to
Clown WM saicht:

> Der Platz kann ebenso eine Konstante enthalten.

ROFL...

> Ist der Ausdruck ∀n ∈ ℕ_def ∃ω: ω > n in Deiner Matheologie verboten?
> In gewöhnlicher Logik nicht.

ROFL... zum Totlachen...

Fritz Feldhase

unread,
May 20, 2022, 4:24:26 PM5/20/22
to
Nein, LUSTIG ist das schon lange nicht mehr (m. E.).

Tom Bola

unread,
May 20, 2022, 4:34:49 PM5/20/22
to
Fritz Feldhase schrieb:
Die mit "Matheologie" verbundene Beleidigung nicht, nein.

Fritz Feldhase

unread,
May 20, 2022, 4:51:03 PM5/20/22
to
On Friday, May 20, 2022 at 9:20:58 PM UTC+2, Transfinity wrote:
> Ralf Bader schrieb am Freitag, 20. Mai 2022 um 21:06:50 UTC+2:
> > On 05/20/2022 08:25 PM, Ganzhinterseher wrote:
> > >
> > > Jede natürliche Zahl ist kleiner als omega. omega ist größer als jede natürliche Zahl.

Ja, Mückenheim, das wird (im Kontext der Mengenlehre) "in kanonischer Weise" SO formalisiert:

∀n(n ∈ ℕ -> n < ω)

bzw.

∀n(n ∈ ℕ -> ω > n).

Oder - etwas komprimiert - auch so:

∀n ∈ ℕ: n < ω

bzw.

∀n ∈ ℕ: ω > n .

In der Tat ist in diesem Zusammenhang eine unzulässige "Quantorvertauschung" gar nicht möglich, weil hier nur EIN Quantor vorkommt.

Selbst Chuck Norris könnte das also IN DIESEM FALL nicht vollbringen.

Da Sie Sich aber hier unbedingt zum Affen machen wollen, glauben Sie schreiben zu müssen:

> > > ∀n ∈ ℕ ∃ω: ω > n
> > > ∃ω ∀n ∈ ℕ: ω > n

Sinnloser Quatsch.

Man kann hier Ralf nur zustimmen: "Mit der offenbaren Verwendung der Konstanten ω am Platz der Variablen in einem Quantorenausdruck bestätigen Sie erneut, daß Sie für Mathematik zu blöde sind und nur Scheißdreck daherschwafeln."

Tatsächlich MEINEN Sie natürlich etwas anderes als das, was sie konkret geschrieben haben. Nämlich z. B.:

(1) ∀n ∈ ℕ ∃o ∈ ORD: o > n

bzw.

(2) ∃o ∈ ORD: ∀n ∈ ℕ: o > n

BEIDES ist zwar richtig (wie man beweisen kann), allerdings kann man NICHT von (1) durch "Quantorvertauschung" auf (2) "schließen", Sie Vollpfosten. Man muss also (2) auf andere Weise zeigen/beweisen.

Vielleicht kapieren Sie es ja noch einmal vor ihrem Ableben:

DIE QUANTORVERTASCHUNG IST EINE UNZULÄSSIGE "SCHLUSSWEISE". SIE KANN (DARF) ALSO NICHT VERWENDET WERDEN, UM IRGENDWAS ZU BEWEISEN.

Aber ich denke, es hat wohl keinen Sinn, Ihnen diese einfachen Sachverhalt, den jeder Student im ersten Semester versteht (verstehen muss!), näherbringen zu wollen.

> > Mit der offenbaren Verwendung der Konstanten ω am Platz der Variablen bestätigen Sie erneut, daß Sie für Mathematik zu blöde sind und nur Scheißdreck daherschwafeln.
> >
> Der Platz kann ebenso eine Konstante enthalten.

Nein, kann er nicht. Denn dann ist der so gebildete Ausdruck keine wff mehr. Also kein zulässiger Ausdruck im Kontext der FOPL (Prädikatenlogik erster Stufe).

> Ist der Ausdruck "∀n ∈ ℕ ∃ω: ω > n" in [der Mathematik/Mengenlehre] verboten?

Ja, das ist er. Da "ω" (in mengentheoretischen Kontexten) üblicherweise eine _Konstante_ ist, die die kleinste Ordinalzahl bezeichnet.

> In gewöhnlicher Logik nicht.

Doch, doch. Insofern die FOPL (Prädikatenlogik erster Stufe) üblicherweise als "die gewöhnliche Logik" in der Mathematik angesehen wird. Der Ausdruck wäre aber auch in einer HOL unzulässig, da im Zusammenhang mit Quantoren generell keine Konstanten zulässig sind.

Natürlich mag das in Ihrem Wahnsystem anders sein - muss wohl so sein.

Geh' scheißen, Mückenheim!
It is loading more messages.
0 new messages